Sunteți pe pagina 1din 72

253

Contraception

38–1. Which o the ollowing is considered long-acting 38–6. The Skyla IUD can be di erentiated rom the other
reversible contraception (LARC)? FDA-approved levonorgestrel-releasing IUD by
a. Copper intrauterine device which o the ollowing characteristics?
b. Depot medroxyprogesterone acetate a. White, T-shaped rame
c. Combination hormonal contraceptive patch b. Silver ring atop thestem
d. All o the above c. Copper bracelets on the lateral T arms
d. Single braided brown string attached to the
38–2. What is the goal o the United States Medical T- rame base
Eligibility Criteria (US MEC) published by the
Centers or Disease Control and Prevention? 38–7. Contraceptive e icacy with this method is believed
a. Guide contraceptive selection or women with to result rom which o the ollowing?
comorbidities
b. Provide legal criteria or contraception provision
to minors
c. Provide inancial criteria or Medicaid
contraception eligibility
d. None o the above

38–3. With typical use, which o the ollowing


contraceptive methods has the highest ailure rate
within the irst year o use?
a. Withdrawal
b. Spermicides
c. Male condom
d. Progestin-only pills

38–4. O U.S. Food and Drug Administration (FDA)-


approved intrauterine devices (IUDs), hormone-
eluting devices release which o the ollowing?
a. Norgestimate
b. Etonogestrel
c. Norethindrone
d. Levonorgestrel a. Spermicidal action
b. Ovulation inhibition
38–5. O U.S. FDA-approved intrauterine devices, c. Cervical mucus thickening
which o the ollowing has the longest duration o
contraceptive e icacy?
d. All o the above
a. Paragard, copper intrauterine device
b. Skyla, levonorgestrel-releasing intrauterinedevice
c. Mirena, levonorgestrel-releasing intrauterine
device
d. Skyla and Mirena have equivalent durations o
e icacy
254 The Puerperium

38–8. During insertion, which o the ollowing increases 38–10. Your patient complains o lower abdominal ache in
this IUD-related complication? the months ollowing placement o her IUD. During
examination, no IUD strings are seen. Which o the
ollowing is the most appropriate nextmanagement step?
a. Breast eeding
a. Obtain KUB radiograph
b. Insertion during menses
b. Reexamine her ollowing her next menses
c. Midplane uterine position
c. Per orm β -human chorionic gonadotropin
d. None o the above (β -hCG) assay
d. Attempt intrauterine IUD retrieval by means o
an IUD hook or orceps

38–11. The patient in Question 38–10 has a negative


38–9. Compared with the copper IUD, this intrauterine β-hCG test result. Which o the ollowing is the best
device o ers which o the ollowing long-term next management step?
advantages? a. Diagnostic laparoscopy
b. Diagnostic hysteroscopy
c. Transvaginal sonography
a. Lower rates o menorrhagia d. Magnetic resonance imaging
b. Longer duration o e ective use
c. Lower rates o ovarian cyst ormation 38–12. Her sonographic images are shown here, and an
arrow indicates a sagittal view o the IUD. Which o
d. None o the above
A the ollowing is the best next clinicalstep?

a. Hysterectomy
b. Diagnostic laparoscopy
c. Dilation and curettage
d. Operative hysteroscopy

Contraception 255

38–13. Which o the ollowing statements regarding IUD 38–15. An asymptomatic woman with 8 weeks o
use and in ection are correct? amenorrhea is ound to have a positive pregnancy
a. Prophylaxis with doxycycline 100 mg orally in a test result, strings o her Mirena IUD visible at the
single dose is required prior to IUD insertion. cervical os, and sonographic indings shown here.
B No adnexal pathology was seen sonographically. She
b. Endocarditis prophylaxis is required or those
desires to continue the pregnancy i possible. What
with mitral valve prolapse prior to IUD insertion.
is the most appropriate managementor her?
c. Negative Neisseria gonorrhoeae and Chlamydia B
trachomatis test results are required prior to IUD
insertion. a. IUD removal
d. None o the above b. IUD removal and dilatation and curettage
c. IUD removal and 10-daycourse o doxycycline
38–14. This photograph shows placental membranes
ollowing a term delivery. Antenatally, this
d. Initiate olate and schedule irst prenatal care visit
or 2 weeks
pregnancy was at increased risk or which o the
ollowing?

a. Fetal mal ormations


b. Intrauterine in ection
c. Fetal-growth restriction
d. All o the above A
256 The Puerperium

38–16. I an intrauterine device is le t in situ during 38–20. Which o the ollowing is a suspected method o
pregnancy, what is the risk o this pregnancy e icacy or progestin-containing contraceptive
complication? implants?
a. Endometrial atrophy
a. 10% b. Ovulation inhibition
b. 25% c. Cervical mucus thickening
c. 50% d. All o the above
d. 75%
38–21. According to the US MEC, in addition to pregnancy
and current breast cancer, which o the ollowing
38–17. Which o is true regarding the timing o IUD are absolute contraindications to progestin-releasing
insertion? implants?
a. May be inserted immediately ollowing vaginal a. Prior cervical cancer and prior deep-vein
delivery thrombosis
b. May be inserted during the menstrual b. Prior ectopic pregnancy and prior deep-vein
cycle ollicular phase once pregnancy is thrombosis
excluded c. Prior cervical cancer, prior ectopic pregnancy, and
c. May be inserted immediately ollowing dilation diagnosed depression
and curettage or irst-trimester abortion d. None o above
d. All o the above
38–18. Which hormone is released rom the contraceptive 38–22. Rates o which o the ollowing are increased with
implant currently marketed in the United States? use o extended cycle hormonal contraception
a. Norgestimate compared with traditional cyclic hormonal
contraception use?
b. Etonogestrel
c. Norethindrone a. Headaches
d. Levonorgestrel b. Escape ovulation
c. Endometrial cancer
38–19. A new patient requests removal o her Implanon d. Unpredictable bleeding
implant. You are unable to locate the device by
palpation. What is the best next management step? 38–23. Your patient calls you on Monday stating that she
a. Sonography missed taking her birth control pills during the
b. Fluoroscopy weekend. All EXCEPT which o the ollowing are
true counseling statements?
c. Radiography
a. Take three pills today and then inish the current
d. Computed-tomography scanning
pill pack
b. Take one pill today, inish the pill pack, but add
a barrier method until her next menses
c. Stop pills, use a barrier method until the next
menses, then restart a new pack on irst day o
menses
d. Discard current pill pack, start new pill pack
today, but add an additional barrier method or
7 days

38–24. Drugs whose e icacy may be diminished by


combination oral contraceptive pill use include
which o the ollowing?
a. Lisinopril
b. Macrodantin
c. Acetaminophen
d. Metoclopramide
Contraception 257

38–25. Which o the ollowing is not a physiologic e ect 38–30. With transdermal contraceptive patch use, the patch
exerted by the estrogen component o combination should be replaced how requently?
hormonal contraceptives?
a. Elevated serum ibrinogen levels a. Daily
b. Lowered serum ree testosterone levels b. Weekly
c. Elevated serum ollicle-stimulating hormone c. Twice weekly
levels
d. Every 3 weeks
d. Lowered serum low-density-lipoprotein (LDL)
cholesterol levels
38–31. With transdermal contraceptive patch use,
38–26. I prescribing combination oral contraceptive pills to contraceptive ailure may occur in which o the
a woman with hypertension, which clinical criteria ollowing situations?
should be met?
a. Patient weight > 90 kg
a. Nonsmoker
b. Concurrent penicillin antibiotic use
b. Younger than 35 years
c. Patch worn while in hot tub or sauna
c. Hypertension well controlled
d. All o the above
d. All o the above
38–32. With this method o contraception, a new ring is
38–27. Which o the ollowing is not an absolute used how requently?
contraindication to initiation o combination oral
contraceptive pills?
Reproduced with permission rom Cunningham FG, Leveno KJ, Bloom SL, et al (eds): Con-
a. Thrombotic disorders traception. In Williams Obstetrics, 24th ed. New York, McGraw-Hill, 2014, Figure 38-5.
b. Two weeks postpartum
c. Migraines with ocal neurologic de icits a. Daily
d. Uncomplicated systemic lupus erythematosus b. Weekly
with negative antiphospholipid antibody testing c. Monthly
d. Every 3 months
38–28. The relative risk o which o the ollowing cancers
has been associated with current combination oral
contraceptive pill use?
a. Ovarian
b. Melanoma
c. Cervical
d. Endometrial

38–29. A proven bene it o combination oral contraceptive


pills includes which o the ollowing?
a. Increased bone density
b. Improved cognitive memory
c. Lowered total cholesterollevels
d. Decreased rates o thromboembolism
258 The Puerperium

38–33. Compared with combination oral contraceptive pills, 38–38. The incidence o which o the ollowing is increased
the vaginal contraceptive ring is associated with with diaphragm use?
higher rates o which o the ollowing? a. Cystocele
a. Toxic shock syndrome b. Cervicitis
b. Contraceptive ailure c. Urinary tract in ection
c. Cervical intraepithelial neoplasia d. Pelvic in lammatorydisease
d. None o the above
38–39. Which o the ollowing is NOT a variation o
38–34. Risks and bene its o depot medroxyprogesterone periodic abstinence as a amily planning method?
acetate (DMPA) use or longer than 2 years should a. Withdrawal method
be evaluated care ully, especially in which o the b. Cervical mucus method
ollowing patients?
c. Calendar rhythm method
a. Hypertensive women
d. Temperature rhythm method
b. Perimenopausal women
c. Women with a amily history o breast cancer
38–40. Compared with latex diaphragms, this product o ers
d. Women with a history o simple endometrial which advantages?
hyperplasia
a. Greater e icacy
38–35. Potential side e ects o DMPA commonly may b. Prescription not required
include which o the ollowing? c. Not associated with subsequent vaginitis
a. Amenorrhea d. Not associated with toxic shock syndrome
b. Irregular uterine bleeding
c. Delayed return o ertility ollowing DMPA 38–41. Current methods o emergency contraception
cessation include all EXCEPT which o the ollowing?
d. All o the above a. Ulipristal acetate
b. Levonorgestrel-releasing IUD
38–36. Your patient has diabetes and hypertension but c. Levonorgestrel-containing pills
pre ers to use “pills” or contraception. She declines
d. Combination oral contraceptive pills
an intrauterine device and barrier methods. She is
considering a progestin-only contraceptive and avors
progestin-only pills. You counsel her regarding the
advantages and which o the ollowing disadvantages
o progestin-only pills compared with combination
oral contraceptives?
a. Higher ailure rate
b. Higher rate o irregular bleeding
c. Higher relative rate o ectopic pregnancy with
method ailure
d. All o the above

38–37. Latex condom e icacy is enhanced by which o the


ollowing?
a. Reservoir tip
b. Oil-based spermicide
c. Concurrent emale condom use
d. All o the above
260

Sterilization

39–1. Your patient is a 26-year-old G3P2 who desires 39–4. Although highly e ective and shown here, which
permanent sterilization ollowing her upcoming method is an uncommonlyused technique or
delivery. Her past medical and surgical histories puerperal sterilization?
are unremarkable. During your counseling session
you quote ailures rates or puerperal sterilization; Reproduced with permission rom Hof man BL: Surgeries or benign gynecologic condi-
you discuss alternativemethods o contraception, tions. In Schorge JO, Schaf er JI, Halvorson LM, et al (eds) Williams Gynecology. New
including the advantages o vasectomy; and York, McGraw-Hill, 2008, Figure 41-24.5.

you discuss possible operative and anesthesia


complications. Which other preoperative counseling a. Uchida
points regarding emale tubal sterilization are true? b. Pomeroy
a. Ease o sterilization reversal c. Parkland
b. Higher risk o later menstrual irregularities d. Modi ied Pomeroy
c. Choice o either an abdominal or a hysteroscopic
approach or puerperal sterilization 39–5. Sterilization in the puerperium is typically per ormed
d. None o the above using which o the ollowing anesthetic methods?
a. Spinal anesthesia
39–2. Which o the ollowing aspects o normal b. General anesthesia
postpartum maternal anatomy are not advantageous c. Incision in iltration
or puerperal sterilization? d. Transversus abdominis plane block
a. Noninvoluted uterus
b. Lax anterior abdominal wall
c. Engorged mesosalpinx vessels
d. All o the above

39–3. Outcomes associated with per orming puerperal


tubal sterilization the morning a ter vaginal delivery
include which o the ollowing?
a. Increases the risk o anesthesia-related
complications
b. Provides longer opportunity to assess neonatal
well-being
c. Increases the risk o postpartum hemorrhage
complicating the postoperative course
d. All o the above
Sterilization 261

39–6. During puerperal sterilization and ollowing 39–10. With this method o interval sterilization, which o
peritoneal cavity entry, the small bowel and the ollowing energy sources is pre erred?
omentum continue to dri t into your operative ield.
All EXCEPT which o the ollowing techniques can
improve visualization? a. Harmonic ultrasound
a. Enlarge the incision b. Bipolar electrosurgical coagulation
b. Use a wider retractor c. Unipolar electrosurgical coagulation
c. Place patient in reverse Trendelenburg position d. Nd:YAG (neodymium-doped yttrium aluminum
d. Pack the omentum cephalad with an opened garnet) laser
4 × 4 gauze sponge whose extraabdominal end
has been tagged with a hemostat 39–11. Which o the ollowing is true o nonpuerperal
emale tubal sterilization per ormed in the United
39–7. Prior to ligation, the allopian tube is most reliably States?
identi ied by which o its ollowing attributes? a. Commonly per ormed via minilaparotomy
a. Fimbria b. Commonly per ormed using general anesthesia
b. Pronounced vascularity c. Typically requires overnight hospitalization
c. Round tubular structure d. None o the above
d. Location posterior to the uteroovarianligament
39–12. O available methods, which o the ollowing is
39–8. With the Parkland method o emale tubal commonly used or nonpuerperal tubal sterilization
sterilization, it is recommended that approximately in the United States?
what length o tubal segment be excised to allow a. Laparoscopic clip application
adequate separation o tubal stumps? b. Laparoscopic supracervical hysterectomy
c. Distal salpingectomy via posterior colpotomy
a. 0.5 cm
d. Hysteroscopic intratubal quinine instillation
b. 1 cm
c. 2 cm
d. 4 cm

39–9. The success o the Pomeroy procedure relies upon


the use o what type o suture ligature?
a. Plain gut
b. Chromic gut
c. Synthetic polyglactin braided
d. Synthetic polydioxanone mono ilament
262 The Puerperium

39–13. This image shows the cumulative probability o 39–15. This surgical specimen is the result o which
pregnancy per 1000 procedures by ive methods intraoperative event during puerperal sterilization?
o tubal sterilization using data rom the U.S.
Collaborative Review o Sterilization (CREST)
study. When counseling your patient regarding
a. Incidental appendectomy
puerperal bilateral midsegment salpingectomy, which
o the ollowing long-term ailure rates per procedure b. Correct allopian tube ligation
is most accurate? c. Incorrect round ligament ligation
d. Incorrect uteroovarian ligament ligation
a.1 in 150
+ 39–16. With Filshie clip application or nonpuerperal
b.1 in 650 emale tubal sterilization, which o the ollowing
c. 1 in 1100 is a possible reason or method ailure with this
d. 1 in 2000 technique?
a. Tubal stump istula
39–14. Puerperal sterilization ailures result most commonly b. Electrosurgical circuit ailure
rom which o the ollowing? c. Intercourse too soon a ter the procedure
a. Ligation o incorrect structure d. Surgery scheduled in the ollicular phase
b. Postoperative acute salpingitis
c. Use o absorbable suture or ligation 39–17. I pregnancy occurs ollowing bilateral bipolar
d. Intercourse too soon ollowing ligation tubal electrosurgical coagulation, what is the rate o
ectopic tubal implantation?
a. 10%
b. 30%
c. 50%
d. 75%
Sterilization 263

39–18. Your patient is a 37-year-old G4P4 who underwent 39–21. Your 37-year-old patient underwent bilateral tubal
puerperal sterilization 14 months ago and now bipolar coagulation or sterilization 7 years ago.
presents or her annual examination. She has a Review o her operative report reveals that you
history o polycystic ovarian syndrome with irregular coagulated three contiguous spots along a 3-cm
menses and notes that her last menses was 7 weeks length that incorporated most o the tube’sisthmic
ago. Appropriate management should irst be which portion. Now in a new relationship, she presents
o the ollowing? desiring counseling regarding reestablishment
a. Transvaginal sonographic examination o ertility. Which o the ollowing is a correct
b. Serum human chorionic gonadotropin level counseling statement regarding tubal reanastomosis?
assessment a. Highest pregnancy rates ollow ampullary-to-
c. Initiation o oral contraceptives or menstrual isthmic reanastomosis.
regulation b. Almost 10 percent o women who conceive
d. Medroxyprogesterone 10 mg daily or 10 days to ollowing reanastomosis will have an ectopic
initiate a withdrawal menses pregnancy.
c. Reanastomosis a ter tubal electrosurgical
39–19. Which othe ollowing conditions is increased in coagulation has higher pregnancy rates than
women ollowing this procedure compared with reanastomosis ollowing Filshie clip application.
those not undergoing this surgery? d. All o the above

39–22. This device achieves sterilization by which o the


a. Risk o ovarian cancer ollowing methods?
b. Incidence o menorrhagia
a. Lies within the cervical canal to secrete
c. Risk o pelvic in lammatory disease spermicide
d. None o the above b. Wraps around the allopian tube to occlude the
lumen
39–20. Studies support that tubal ligation is a risk or which c. Is placed within the uterine cavity to agglutinate
o the ollowing psychological or sexual side e ects? the endometrium
a. Regret d. Is placed within the allopian tube ostia to
b. Decreased libido promote occlusive tissue ingrowth
c. Decreased sexual satis action
d. All o the above 39–23. According to Food and Drug Administration
recommendations, which o the ollowing imaging
procedures is per ormed a ter the Essure method o
tubal sterilization to document proceduresuccess?
a. Hysterosalpingography 6 weeks a ter surgery
b. Saline in usion sonography 3 months a ter
surgery
c. Three-dimensional transvaginal sonography
8 weeks a ter surgery
d. None o the above
264 The Puerperium

39–24. All EXCEPT which o the ollowing are true 39–29. Most vasectomy ailures occur during the irst year
regarding the Essure method o tubal sterilization? ollowing the procedure. The cumulative ailure rate
a. Requires tissue ingrowth or method success per 1000 procedures at years 2, 3 and 5 is stable and
approximates which o the ollowing?
b. Requires hysterosalpingography 3 months
postprocedure a. 10
c. Requires proximal tubal thermal injury to incite b. 50
tissue ingrowth c. 80
d. Requires alternative method o contraception d. 125
until tubal occlusion is documented
39–30. Vasectomy ailures may result rom which o the
39–25. Pregnancy ollowing the Essure method o ollowing?
transcervical tubal sterilization may result rom a. Recanalization
which o the ollowing?
b. Incomplete surgical occlusion
a. Insert expulsion c. Intercourse too soon a ter the procedure
b. Misinterpretation o hysterosalpingogram d. All o the above
c. Noncompliance with required
hysterosalpingogram
39–31. A long-term complication ollowing vasectomy
d. All the above includes which o the ollowing?
a. Regret
39–26. During vasectomy, which o the ollowing structures
b. Atherogenesis
is ligated?
c. Testicular cancer
a. Epididymis
d. All o the above
b. Spermatic cord
c. Ductus de erens
39–32. Pregnancy rates a ter vasectomy reversal increase
d. E erent ductile
with all EXCEPT which o the ollowing?
a. Microsurgical technique
39–27. Compared with vasectomy, which o the ollowing is
b. Younger emale partner age
higher with emale tubal sterilization?
c. Longer time duration rom vasectomy to reversal
a. Cost
b. Failure rate d. Normal sperm quality noted during reversal
procedure
c. Surgical complication rate
d. All o the above

39–28. To avoid conception ollowing vasectomy, an


alternative orm o contraception should be used
until semen analysis documents aspermia. Complete
sperm expulsion rom the reproductive tract takes
approximately how long?
a. 1 week
b. 4 weeks
c. 8 weeks
d. 12 weeks
320

Obesit

48–1. Which of the following best describes obesity in the 48–3. A 22-year-old G1P0 presents for prenatal care at 8
United States? weeks’ gestation. She has a BMI of 26. The National
a. Endemic Institutes of Health classifies this patient as which of
b. Epidemic the following?
c. Pandemic a. Normal
d. Syndemic b. Overweight
c. Obese–class 1
48–2. Body mass index (BMI) is defined as which of the d. Obese–class 2
following?
a. Height in inches divided by weight in pounds 48–4. A 19-year-old G1P0 presents for her first prenatal
squared. care appointment. She weighs 270 pounds and is
67 inches tall. Using the figure provided, what is the
b. Weight in pounds divided by height in inches
patient’s BMI?
squared.
a. < 18.5
c. Weight in kilograms divided by height in meters
squared. b. 19–25
d. Height in meters divided by weight in kilograms c. 25–30
squared. d. > 30.0
Obesity 321

48–5. Based on this figure, which of the following 48–8. A 35-year-old woman presents to your office
statements is true? for preconceptional counseling. She has chronic
hypertension that is controlled with medication. Her
blood pressure today is 140/90 mm Hg, and her
a. The prevalence of obesity increases with age. waist circumference is 36 inches. A fasting glucose
b. Race is not a factor in the prevalence of female level is 115 mg/dL, high-density lipoprotein (HDL)
obesity. concentration is 60 mg/dL, and triglyceride level is
c. Obesity is more common in Hispanic women 145 mg/dL. How many criteria does she meet for
than in black, non-Hispanic women. metabolic syndrome?
d. Obesity is more common in white, non-Hispanic a. 1
women than in Hispanic women. b. 2
c. 3
d. 4
48–6. Which of the following statements regarding
adiponectin is not true? 48–9. Which of the following is not part of the metabolic
a. It enhances insulin sensitivity. syndrome?
b. It blocks the hepatic release of glucose. a. Diabetes
c. It contributes to the development of b. Dyslipidemia
hypertension. c. Hypertension
d. It has cardioprotective effects on circulating d. Polycystic ovaries
plasma lipids.
48–10. Nonalcoholic fatty liver disease (NAFLD)
48–7. Which of the following does not contribute to contributes what percentage of chronic liver disease
insulin resistance? cases in Western countries?
a. Leptin a. 1%
b. Resistin b. 10%
c. Adiponectin c. 25%
d. Tumor necrosis factor-alpha (TNF-α) d. 50%

48–11. Rates of which of the following are increased in


obese women?
a. Infertility
b. Recurrent miscarriage
c. First-trimester pregnancy loss
d. All of the above

48–12. A 25-year-old G2P1 at 12 weeks’ gestation


presents to your office for prenatal care. She has a
BMI of 27. Based on the Institute of Medicine’s
recommendations, how much weight should she
gain thispregnancy?
a. None
b. 11–20 pounds
c. 15–25 pounds
d. 30–40 pounds

48–13. What percentage of women having their first baby


gain more weight than recommended?
a. 1%
b. 10%
c. 25%
d. 75%
322 Medical and Surgica l Complications

48–14. Which of the following adverse pregnancy effects is 48–20. Serum levels of which of the following are increased
not of increased prevalence in overweight women in obese pregnant women?
compared with women of normal BMI? a. Insulin
a. Preeclampsia b. Interleukin-6
b. Urinary infection c. C-reactive protein
c. Gestational diabetes d. All of the above
d. Emergency cesarean delivery
48–21. Which of the following is the highest-ranking
48–15. Incidence of which of the following medical modifiable risk factor for stillbirth?
problems is not increased with increasing BMI? a. Obesity
a. Macrosomia b. Smoking
b. Preeclampsia c. Diabetes
c. Gestational diabetes d. Maternal age
d. All have increased incidence
48–22. The rate of which of the following is not increased
48–16. You have two similar pregnant women in your in fetuses of obese pregnant women?
practice. Both are 25 years old, primiparous, a. Stillbirths
and free of diabetes and hypertension. One has a b. Heart defects
BMI of 25, and the other has a BMI of 35. What
c. Renal agenesis
is the difference in their chances of developing
preeclampsia? d. Neural-tube defects
a. Equal risk
b. A 3% chance versus a 12% chance 48–23. Which of the following has the strongest influence
on the prevalence of the condition depicted in this
c. A 10% chance versus a 25% chance photograph?
d. A 10% chance versus a 40% chance
a. Prepregnancy BMI
48–17. In the study by Weiss and colleagues from 2004, b. Gestational diabetes
what was the cesarean delivery rate for morbidly c. Gestational weight gain
obese women?
d. Pregestational diabetes
a. 10%
b. 21% 48–24. Which of following contributes to childhood
c. 47% obesity?
d. 62% a. Maternal obesity
b. Maternal gestational diabetes
48–18. Compared with normal-weight patients, obese c. Maternal-child environment subsequent to birth
women have increased rates of all EXCEPT which of d. All of the above
the following?
a. Hypertension
b. Wound infection
c. Vaginal delivery
d. Gestational diabetes

48–19. Which of the following statements regarding obesity


and pregnancy is not true?
a. Obese women are more likely to breast feed.
b. Postpartum depression rates are increased in
obese women.
c. Obese women have greater weight retention
1 year after delivery.
d. Second-trimester dilatation and evacuations take
longer in obese women.
Obesity 323

48–25. During the prenatal care of an obese woman, which 48–27. Regarding cesarean delivery in obese women, which
of the following commonly collected measures will of the following should be done to reduce the
likely be LEAST useful in patient management? chances of wound disruption?
a. Fundal height a. Vertical skin incision
b. Urine protein b. Weight loss during the pregnancy
c. Blood pressure c. Closure of the subcutaneouslayer
d. Cervical dilatation d. All of the above

48–26. When performing a cesarean delivery on the obese 48–28. Which of the following is not recommended by the
pregnant woman, which of the following is not an American College of Obstetricians and Gynecologists
advantage of the incision depicted in this figure? to lower thromboembolic complications in the obese
parturient?
a. Hydration
a. Provides the least intervening tissue b. Early mobilization
b. Permits access to the lower uterine segment c. Compression stockings
c. Greatly reduces the chances of wound infection d. Full anticoagulation with heparin
d. Prevents moisture from collecting on the incision
48–29. Which of the following complications is more likely
in a pregnant woman who has undergone bariatric
surgery?
a. Diabetes
b. Preeclampsia
c. Large-for-gestational age neonate
d. Small-for-gestational age neonate

48–30. Which of the following complications is more


common after Roux-en-Y gastric bypass than after
gastric banding?
a. Hypertension
b. Cesarean delivery
c. Gestational diabetes
d. Small-for-gestational age newborn

48–31. A 30-year-old G3P2 at 14 weeks’ gestation presents


for prenatal care. She reports that she had bariatric
surgery 2 years ago, losing 100 pounds. You
correctly counsel her which of the following?
a. She is at increased risk for diabetes.
b. She is at increased risk for hypertension.
c. She does not need to see her bariatric doctor
during the pregnancy.
d. She needs to be assessed for vitamin deficiencies,
and she may need vitamin B12, vitamin D, or
calcium supplementation.

48–32. When caring for a pregnant woman who has


undergone bariatric surgery, vigilance for which of
the following is essential?
a. Appendicitis
b. Bowel obstruction
c. Acute cholecystitis
d. Gastroesophageal reflux disease
325

CHAPTER00

Cardiovascular Disorders

49–1. What percent o pregnancies are complicated by 49–6. All EXCEPT which o the ollowing are
heart disease? electrocardiogram changes seen in normal
a. 0.1% pregnancy?
b. 1% a. Increased voltage
c. 4% b. 15-degree le t axis deviation
d. 8% c. Premature atrial contractions
d. ST segment changes in in erior leads
49–2. Which o the ollowing parameters increases by
40 percent in pregnancy? 49–7. A pregnant woman with preexisting cardiac disease is
a. Heart rate com ortable at rest but cannot stand up to brush her
teeth without experiencing chest pain. What is her
b. Cardiac output New York Heart Association classi ication?
c. Mean arterial pressure a. I
d. Le t ventricular stroke work index b. II
49–3. Which aspect o cardiac physiology does not change c. III
in pregnancy? d. IV
a. Mean arterial pressure
49–8. Excluding those associated with Mar an syndrome,
b. Pulmonary vascular resistance which congenital heart lesions carry the greatest risk
c. Le t ventricular contractility o heritability?
d. Le t ventricular stroke work index a. Aortic stenosis
b. Pulmonary stenosis
49–4. Which o the ollowing is never a normal inding in
pregnancy? c. Tetralogy o Fallot
a. Dyspnea d. Coarctation o the aorta
b. Systolic murmur 49–9. A pregnant patient at 13 weeks’ gestation has known
c. Diastolic murmur heart disease. All EXCEPT which o the ollowing
d. Exercise intolerance are predictive o poor outcomes in pregnancy?
a. Prior stroke
49–5. When encountered in an obstetrical patient, which b. Cardiac symptoms at rest
o the ollowing are considered expected variations
rom the nonpregnant state?
c. Ejection raction o 30%
a. Larger cardiac silhouette seen on chest radiograph d. Mitral valve area o 2.5 cm2
b. Tricuspid regurgitation seen during 49–10. Which o the ollowing statements is true regarding
echocardiography antepartum and intrapartum care o patients with
c. Increased le t atrial end-diastolic dimensions seen cardiovascular disease?
during echocardiography a. Vaginal delivery is pre erred.
d. All o the above b. Spinal blockade is the recommended anesthetic.
c. These patients should avoid pneumococcal
vaccination.
d. Invasive monitoring with pulmonary artery
catheter is required.
326 Medical and Surgica l Complications

49–11. A patient with prior valve replacement heart surgery 49–14. I a patient requires cardiac bypass or valve
presents or prenatal care. Her chest radiograph replacement during pregnancy, which o the
lateral view is shown here and arrows point to her ollowing can sheexpect?
St. Jude valve. With mechanical valves, what is the a. Fetal death rate o 5%
maternal mortality rate in pregnancy? b. Minimal miscarriage risk
a. 0.1–0.4% c. Maternal mortality rate o 20%
b. 3–4% d. Maternal mortality rate similar to nonpregnant
c. 7–9% women
d. 12–15%
49–15. A pregnant patient with mitral stenosis requires
percutaneous transcatheter balloon dilatation o the
mitral valve. What can you tell the patient as part o
49–12. In a pregnant woman with a mechanical heart valve, your presurgical counseling?
which o the ollowing anticoagulation regimens
a. It is pre erred to open surgical repair.
would be considered inadequate or prevention o
thromboembolism involving the prosthesis? b. Le t atrial pressures will improve as the mitral
valve area increases.
a. War arin
b. Low-dose un ractionated heparin c. Balloon dilatation is success ul in more than 90
percent o procedures.
c. Adjusted-dose un ractionated heparin
d. All o the above
d. Adjusted-dose low-molecular-weight heparin
49–16. A patient who had a heart transplant as a child is
49–13. Which o the ollowing anticoagulants is not now considering pregnancy. What complications can
compatible with breast eeding? she expect duringpregnancy?
a. War arin a. A tissue rejection risk less than5%
b. Un ractionated heparin b. A high likelihood o vaginal delivery
c. Low-molecular-weight heparin c. A risk o hypertension that approximates 50%
d. All are compatible d. A heart that will not undergo the normal
physiologic changes o pregnancy

49–17. A patient in labor gives a history o mitral stenosis.


She is asymptomatic and currently taking a
beta blocker, but she is not sure why her doctor
recommended it. What complication is the
prophylactic beta blocker trying to prevent in these
patients?
a. Irregular heart rhythm
b. Mural thrombus ormation
c. Tachycardia leading to pulmonary edema
d. Le t ventricular hypertrophy and dilatation
Ca rdiovascular Disorders 327

49–18. The patient whose heart is seen in this image has 49–20. For patients with congenital heart disease, what
systemic lupus erythematosus and has had a prior is the most common adverse cardiovascular event
stroke. The le t atrium (LA) and le t ventricle (LV) encountered in pregnancy?
are identi ied, and the arrows point to nonin ectious a. Arrhythmia
in lammatory vegetations on the mitral lea lets. b. Heart ailure
Which o the ollowing is the likely condition
associated with her mitral insu iciency?
c. Thromboembolic event
d. Cerebrovascular hemorrhage

a. Antiphospholipid antibodies 49–21. Which congenital de ect is associated most o ten


with “paradoxical embolism”?
a. Eisenmenger syndrome
b. Atrial septal de ect
c. Patent ductus arteriosus
d. Ventricular septal de Ca
ect rdiovascular Disorders 329
49–22. A patient with the congenital lesion shown in this
49–31. Which o the ollowing is a threshold or aortic root igure is now pregnant. She has not had surgery to
size above which a patient with Mar an syndrome is correct her lesion, and her hematocrit is 68 volumes
at higher risk o dissection during pregnancy? percent. Her expected risk or pregnancy loss
a. 20 mm approximates which o the ollowing?
b. 30 mm
a. 40%
c. 40 mm
d. 50 mm b. 60%
c. 80%
d. 100%

b. In arction o the papillary muscle 49–32. A 42-year-old gravida at 32 weeks’ gestation with
c. Calci ication o the mitral annulus diabetes presents with chest pain and has an
d. None o the above abnormal electrocardiogram. All EXCEPT which o
the ollowing statements are correct regarding the
evaluation and management o suspected myocardial
49–19. A patient with critical aortic stenosis presents to
in arction in pregnancy?
the labor and delivery unit, and her cervix is 6-cm
dilated. All EXCEPT which o the ollowing are a. The patient should be delivered immediately.
suitable management steps? b. The patient should be started on a beta blocker.
a. Limit activity c. The patient should receive daily low-dose aspirin.
b. Provide narcotic epidural anesthesia d. Troponin levels can be used to aid diagnosis in
c. Per orm pulmonary artery catheterization pregnancy.
d. Decrease cardiac preload by limiting
intravenous luids
328 Medical and Surgica l Complications

49–23. This echocardiography image shows a short-axis view 49–27. An obese primigravida develops acute dyspnea while
o the heart o a emale with Eisenmenger syndrome. A in labor. Her chest radiograph is shown here. What
markedly dilated pulmonary artery (PA) is seen. intervention will most likely resolve her symptoms?
All EXCEPT which o the ollowing are common
underlying de ects that lead to this condition? a. Intravenous urosemide
b. Intravenous hydralazine
a. Atrial septal de ect c. 3 L oxygen by acemask
b. Valvular heart disease d. Therapeutic heparinization
c. Patent ductus arteriosus
d. Ventricular septal de ect
49–28. With in ective endocarditis, what is the most
common causative organism in intravenous drug
49–24. Which World Health Organization classi ication users?
best categorizes patients with idiopathic pulmonary a. Enterococcusspecies
hypertension? b. Staphylococcusaureus
a. I c. Streptococcusviridans
b. II d. Staphylococcusepidermidis
c. III
d. IV 49–29. Which o the ollowing is an adequate intravenous
regimen or prophylaxis o bacterial endocarditis or
49–25. A pregnant patient develops heart ailure without an the at-risk patient in labor who is penicillin allergic?
identi iable underlying cause. I she does not recover a. Ce azolin, 1 g
baseline cardiac unction by 6 months postpartum, b. Ce triaxone, 1 g
which o the ollowing best approximates her 5-year c. Clindamycin, 600 mg
mortality rate? d. All are adequate
a. 5%
b. 20% 49–30. Patients with QT prolongation are at risk o
c. 40% developing which o the ollowing arrhythmias?
d. 80% a. Atrial ibrillation
b. Torsades de pointes
49–26. What is the most common cause o heart ailure c. Wol -Parkinson-White
during pregnancy and the puerperium?
d. Paroxysmal supraventricular tachycardia
a. Obesity
b. Viral myocarditis
c. Valvular heart disease
d. Chronic hypertension with severe preeclampsia
337

CHAPTER00

Pulmonar Disorders

51–1. What per entage pregnant w men have hr ni 51–7. A pregnant patient rep rts asthma sympt ms
asthma? 3 days per week, and she is awakened by these
a.1% appr ximately twi e per m nth.Between
b.4% exa erbati ns, her FEV1 ( r ed expirat ry v lume in
1 se nd) is ≥ 80 per ent predi ted. Her asthma is
c. 14% best lassi ied by whi h the ll wing des ript rs?
d. 20% a. Intermittent
b. Mild persistent
51–2. What happens t vital apa ity in pregnan y?
c. M derate persistent
a. In reases by 20%
d. Very severe persistent
b. De reases by 25%
c. In reases by 40% 51–8. A pregnant patient rep rts asthma sympt ms that
d. De reases by 45% pr mpt daily use her inhaler, and she is awakened
by these sympt ms appr ximately twi eweekly.
51–3. What e e t d es pr gester ne have n tidal v lume? Between exa erbati ns, her FEV1/FVC ( r ed
a. In reases by 20% expirat ry v lume in 1 se nd/ r ed vital apa ity)
b. De reases by 25% is redu ed by 5 per ent. Her asthma is best lassi ied
by whi h the ll wing des ript rs?
c. In reases by 40%
a. Intermittent
d. De reases by 45%
b. Mild persistent
c. M derate persistent
51–4. What happens t arb n di xide pr du ti n in d. Very severe persistent
pregnan y?
a. In reases by20%
b. De reases by 25% 51–9. Whi h the ll wing best hara terizes persistent
asthma?
c. In reases by30%
a. Sympt ms n e daily
d. De reases by 40%
b. FEV1 < 60% predi ted
51–5. What happens t residual v lume in pregnan y? c. S me limitati n n rmal a tivities
a. In reases by20% d. Single, daily use sh rt-a ting β -ag nists r
sympt ms
b. De reases by 20%
c. In reases by40%
51–10. A 25-year- ld G1P0 at 20 weeks’ gestati n
d. De reases by 40% presents with an asthma exa erbati n. Her FEV1 is
50 per ent. Her Pco2 is n rmal, and her Po2 is de
51–6. Whi h the ll wing statements regarding reased. hese values are very n erning and
physi l gi pulm nary hanges in pregnan y is true? likely re le t whi h the ll wing?
a. Basal xygen nsumpti n de reases. a. She is in respirat ry ailure.
b. Ventilati n is in reased by m re requent b. She has a signi i ant metab li a id sis.
breathing. c. A n rmal Pco2 indi ates atigue and early C 2
c. Ventilati n is in reased be ause deeper retenti n.
breathing. d. A etus ann t survive when the m ther’s FEV1
d. Ventilati n is in reased be ause in reased hest dr ps t 50%.
wall mplian e.
338 Medical and Surgica l Complications

51–11. Up t what per entage w men with mild 51–17. Whi h the ll wing d es n t des ribe
r m derate asthma will have an intrapartum the phylline?
exa erbati n? a. Methylxanthine
a. 2% b. Br n h dilat r
b. 20% c. C rti ster id
c. 40% d. Antiin lammat ry agent
d. 60%
51–18. Whi h the ll wing is e e tive r a uteasthma?
51–12. he etal resp nse t maternal hyp xemia is whi h a. Zileut n
the ll wing? b. Cr m lyn
a. De reased ardia utput c. M ntelukast
b. In reased umbili al bl d l w d. Epinephrine
c. De reased systemi vas ular resistan e
d. De reased pulm nary vas ular resistan e 51–19. A 19-year- ld G2P1 at 23 weeks’ gestati n presents
with an asthma exa erbati n. Her FEV1 is 35%, and
51–13. Whi h the ll wing is a sign a p tentially atal she is a ebrile and therwise well. Y u treat her
asthma atta k? with an inhaled β -ag nist treatment and inhaled
a. a hy ardia rti ster ids. A ter three d ses the inhaled
β-ag nist, the patient has an xygen saturati n
b. Central yan sis
94% and an FEV1 45%. Whi h the ll wing
c. Lab red breathing is the best next management step?
d. Pr l nged expirati n
a. rans er t the intensive are unit r status
asthmati us
51–14. Whi h the ll wing statements regarding
b. Dis harge her h me with a sh rt-a ting β -ag nist
pulm nary un ti n testing istrue?
inhaler and s hedule a 2-week ll w-up i e
a. An FEV1 50% rrelates with severe disease. app intment
b. he peak expirat ry l w rate in reases in n rmal c. Admit r additi nal treatment with inhaled
pregnan y. β -ag nists andintraven us rti ster ids and
c. A w man’s baseline pulm nary un ti n tests bserve r respirat ry distress
sh uld be determined when sympt mati . d. Dis harge her h me with β -ag nist and
d. Pulm nary un ti n testing sh uld be r utine in rti ster id nebulizer treatments and with
the management hr ni and a ute asthma. br ad-spe trum antibi ti s r presumed
pneum nia
51–15. he re mmended treatment r mild persistent
asthma sympt ms in ludes a sh rt-a ting β -ag nist 51–20. A pregnant w man is treated r an asthma
and whi h ther agent(s)? exa erbati n at 37 weeks’ gestati n with a regimen
a. N ther agent that in luded systemi rti ster id therapy. She
b. L w-d se inhaled rti ster ids (ICs) presents again at 39 weeks with lab r. Whi h the
ll wing w uld n t be part y ur management
c. L w-d se ICs, and a l ng-a ting β -ag nist
plan?
d. High-d se ICs, and a l ng-a ting β -ag nist
a. Epidural anesthesia, when and i desired by the
patient
51–16. he re mmended treatment r severe persistent
b. Determinati n FEV1 up n admissi n t Lab r
asthma sympt ms in ludes a sh rt-a ting β -ag nist
and Delivery
and whi h ther agent(s)?
c. reatment p stpartum hem rrhage with
a. L w-d se inhaled rti ster ids (ICs) arb pr st tr methamine
b. L w-d se ICs, and a l ng-a ting β -ag nist
d. Hydr rtis ne 100 mg intraven usly every 8 h
c. High-d se ICs, and a l ng-a ting β -ag nist urs during lab r and r 24 h urs a ter
d. High-d se ICs, a l ng-a ting β -ag nist, and ral delivery
rti ster ids
Pulmona ry Disorders 339

51–21. n average, h w l ng d es the ugh a ute 51–25. A 19-year- ld G1P0 at 22 weeks’ gestati n with n
br n hitispersist? signi i ant past medi al hist ry presents with ugh,
a. 1–2 days ever, and hest pain n inspirati n r 3 days.
b. 3–4 days She has a ever 38.5°C, has 22 respirati ns per
minute, and is xygenating well. he patient’s hest
c. 5–7 days
radi graph is pr vided here. he best treatment r
d. 10–20 days this patient is whi h the ll wing?

51–22. Whi h the ll wing is n t a risk a t r r Repr du ed with permissi n r m Cunningham FG, Leven KJ, Bl m SL, et al (eds): Pulm
pneum nia? nary dis rders. In Williams bstetri s, 24th ed. New Y rk, M Graw-Hill, 2014, Figure 51-
3.
a. Asthma
b. Pregnan y
a. Van my in
c. Binge drinking
b. M xi l xa in
d. Chr ni br n hitis
c. Azithr my in
d. Lev l xa in and van my in
51–23. Whi h the ll wing m st mm nly auses
pneum nia?
a. In luenza A
b. Mycoplasma pneumoniae
c. Legionella pneumophila
d. Chlamydophila pneumoniae
A
51–24. Whi h the ll wing statements regarding
pneum nia is true?
a. Chest radi graphy is essential r diagn sis.
b. Chest radi graphy a urately predi ts the
eti l gy.
c. he resp nsible path gen an be identi ied in
m st ases.
d. Sputum ultures are re mmended in all
suspe ted ases pneum nia.

B
340 Medical and Surgica l Complications

51–26. What per entage w men with pneum nia devel p 51–29. A 30-year- ld G2P1 at 15 weeks’ gestati n presents
pleural e usi n? with ugh, ever, and weight l ss. She has re ently
a. 2% immigrated t the United States r m Mexi .
b. 20% Y u send a QuantiFER N- B G ld, whi h yields
a p sitive result. he patient’s hest radi graphy
c. 40% is pr vided bel w. Sputum is p sitive r a id-
d. 60% ast ba illi. Whi h the ll wing is therre t
diagn sis r this patient?
51–27. A 25-year- ld G3P2 at 19 weeks’ gestati n presents
with ever, ugh, myalgias, and nausea r 2 days.
Her tw hildren at h me have als been si k. By Repr du ed with permissi n r m Phan VD, P p ni k JM: uber ul sis. In intinalli JE, Stap
zynski JS, Cline DM, et al (eds): intinalli’s Emergen y Medi ine: A C mprehensive Study
rapid test, the patient is diagn sed with in luenzaA. Guide, 7th ed. New Y rk, M Graw-Hill, 2011, Figure70-2.
he best treatment regimen r this patient is whi h
the ll wing?
a. Sar id sis
a. Amantadine r 5 days b. A tive tuber ul sis
b. Amantadine r 10 days c. Latent tuber ul sis
c. seltamivir r 5 days d. Extrapulm nary tuber ul sis
d. seltamivir and amantadine r 10 days
51–30. Whi h the ll wing is the best management plan
51–28. A 33-year- ld G3P0 at 18 weeks’ gestati n presents r r the patient in Questi n 51–29?
prenatal are. She is br ught t the lini by a. Initiate is niazid
a deputy as she is urrently in ar erated n drug
b. Initiate is niazid and ri ampin
harges. he patient ame t this untry 1 year ag .
She rep rts having previ usly re eived the ba ille c. Initiate is niazid, ri ampin, ethambut l,
Calmette-Guérin (BCG) va ine. By her hist ry, pyrazinamide, and pyrid xine
she has multiple risks r tuber ul sis. Whi h the d. Delay treatment until a ter delivery, then initiate
ll wing is the best initial test r this patient? is niazid, ri ampin, and pyrid xine
a. Chest radi graphy
b. Sputum lle ti n
c. uber ulin skin test
d. Inter er n-gamma release assay (IGRA)
Pulmona ry Disorders 341

51–31. A 38-year- ld G1P0 at 22 weeks’ gestati n presents r 51–32. his image sh ws ive lasses geneti mutati ns
prenatal are. he patient has a tive sar id sis, a e ting a peptide that ultimately un ti ns in
and a ph t graph her hands is pr vided here. epithelial- ell membrane transp rt ele tr lytes.
Regarding sar id sis in pregnan y, y u rre tly hese mutati ns ause whi h the ll wing
unsel her whi h the ll wing? diseases?

Repr du ed with permissi n r m W l K, J hns n RA (eds): T e skin in immune, aut -


immune, and rheumati dis rders. In Fitzpatri k’s C l r Atlas and Syn psis Clini al Dermat l
a. Sar id sis
gy, 6th ed. New Y rk, M Graw-Hill, 2009, Figure 14-51. b. Cysti ibr sis
c. Maple syrup urine disease
a. Perinatal ut mes are p r.
d. Primary iliary dyskinesia
b. Disease pr gressi n in pregnan y is mm n.
c. Severe disease warrants serial determinati n
pulm nary un ti n.
d. All mm n treatment m dalities are abs lutely
ntraindi ated in pregnan y. 51–33. he rganism m st str nglyass iated with ysti
ibr sis is whi h the ll wing?
a. Burkholderia cepacia
b. Staphylococcusaureus
c. Pseudomonasaeruginosa
d. Haemophilusinfluenzae

342 Medical and Surgica l Complications

51–34. Whi h the ll wing statements regarding ysti 51–36. Fetal heart tra ing indingsin ases maternal
ibr sis is true? arb n m n xide p is ning in lude all EXCEPT
a. Disease severity is quanti ied by pulm nary whi h the ll wing?
un ti n studies. a. Elevated baseline
b. When the FEV1 is at least 40%, w men t lerate b. Absen e a elerati ns
pregnan y well. c. Marked beat-t -beat variability
c. he best predi t r pregnan y ut me in d. De reased beat-t -beat variability
w men with ysti ibr sis is nutriti nal status.
d. Diabetes in pregnan y is less likely i the a e ted
w man is h m zyg us r the ∆F508 mutati n.

51–35. he m st mm n ause p is ning w rldwide is


whi h the ll wing?
a. Ri in
b. Cyanide
c. Stry hnine
d. Carb n m n xide
344

Thromboembolic Disorders

52–1. What is the approximate inci ence o 52–6. he mutations that cause antithrombin e iciency
thromboembolic events per pregnancy? are almost always inherite in what ashion?
a. 1/10 a. X-linke ominant
b. 1/100 b. X-linke recessive
c. 1/1000 c. Autosomal ominant
d. 1/10,000 d. Autosomal recessive

52–2. Which o the ollowing statements is correct 52–7. Un erstan ing pregnancy physiology is essential to
regar ing the timing o pregnancy-relate eep-vein iagnose protein S e iciency. Concentrations o
thrombosis (DV ) an pulmonary embolism (PE)? which o the ollowing ecline substantially uring
a. DV an PE are more common antepartum. normal pregnancy?
b. DV an PE are more common postpartum. a. Free protein S
c. DV is more common antepartum, an PE is b. otal protein S
more common postpartum. c. Functional protein S
d. PE is more common antepartum, an DV is d. All o the above
more common postpartum.
52–8. During his irst ay o li e, a seemingly healthy
52–3. All EXCEPT which o the ollowing physiologic neonate who was elivere at term su enly
alterations pre ispose to venous thrombosis in evelops ever an i use skin lesions as shownin
pregnancy? the igure. his is most likely a mani estation o
a. Lower extremity venous stasis what thrombophilic con ition?
b. Decrease plasminogen activity
c. En othelial cell injury at elivery
Repro uce with permission rom Wolf K, Gol smith LA, Katz SI, et al: Fitzpatrick’s
d. Increase synthesis o clotting actors Dermatology in General Me icine, 7th e . New York, McGraw-Hill, 2008, Figure 180-
1C.
52–4. Which o the ollowing risk actors or eveloping
thromboembolism in pregnancy is most important? a. Homozygous prothrombin mutation
a. Cesarean elivery b. Homozygous protein S e iciency
b. Multi etal gestation c. Heterozygous protein C e iciency
c. Postpartum hemorrhage d. Heterozygous actor V Lei en mutation
d. Personal history o thrombosis

52–5. Approximately what percentage o thromboembolic


events that occur uring pregnancy can be attribute
to an inherite or acquire thrombophilia?
a. 1%
b. 15%
c. 50%
d. 75%
Thromboembolic Disorders 345

52–9. Which o the ollowing inherite thrombophilia 52–14. Which o the ollowing thrombophilic con itions
syn romes is most prevalent? have been most consistently associate with a verse
a. Protein S e iciency pregnancy outcomes?
b. Antithrombin e iciency a. Factor V Lei en mutation
c. Factor V Lei en mutation b. Antiphospholipi syn rome
d. Prothrombin G20210A mutation c. Antithrombin mutation
d. Prothrombin G20210A mutation
52–10. Which o the ollowing pregnancy-relate
complications are increase in women who are 52–15. A32-year-ol G2P1 at 29 weeks’ gestation presents
heterozygous carriers o the actor V Lei en or a routine prenatal care visit with no complaints.
mutation? During a routine examination, you note that her
a. Preeclampsia legs have the ollowing appearance. Which o the
b. Placental abruption ollowing statements is true regar ing this con ition
c. Fetal-growth restriction in pregnancy?
d. None o the above

52–11. All EXCEPT which o the ollowing can be causes


o elevate homocysteine levels? a. Most cases are on the right si e.
a. Folate e iciency b. A positive Homans sign is virtually iagnostic.
b. Vitamin D e iciency c. Most cases are locate in the ilio emoral veins.
c. Vitamin B6 e iciency d. Symptoms typically correlate with the egree o
vessel involvement.
d. Methylene-tetrahy ro olate re uctase mutation

52–12. he American College o Obstetricians an


Gynecologists currently recommen s assessment
o women or hyperhomocysteinemia an the
methylene tetrahy ro olate re uctase mutation in
what clinicalsituations?
a. Folic aci e iciency
b. First venous thromboembolism
c. Recurrent venous thromboembolism
d. None o the above

52–13. All EXCEPT which o the ollowing women


illustrate clinical criteria that woul prompt
antiphospholipi syn rome laboratory screening?
a. A33-year-ol G2P1 with a 12-week etal emise.
b. A41-year-ol G2P1 with a history o eclampsia
at 36 weeks in her prior pregnancy.
c. A 26-year-ol G5P1 at 6 weeks’ gestation
with spontaneous abortions in her last three
pregnancies.
d. A 29-year-ol G1P0 at 29 weeks’ gestation
un ergoing in uction or severe etal-growth
restriction.
346 Medical and Surgica l Complications

52–16. he patient escribe in Question 52–15 un ergoes 52–19. his image obtaine uring venographyshows
compression ultrasonography o the lower a illing e ect (arrows) in the popliteal vein.
extremities with the ollowing in ings. he calipers Although this mo ality remains the gol stan ar
elineate a vessel lumen. What next step is most or iagnosing eep-vein thrombosis, it is use
appropriate in her management? in requently or all EXCEPT o the ollowing
reasons?

a. It is invasive.
b. Intravenous contrast is require .
c. It has a low negative-pre ictive value.
d. It increases the risk or proce ure-associate
thrombosis.

52–20. Anticoagulation in pregnancyshoul pre erentially


be accomplishe with which me ication?
a. War arin
b. Dabigatran
a. Venography c. Un ractionate heparin
b. Embolectomy d. Low-molecular-weight heparin
c. Anticoagulation
d. Ventilation-per usion scan

52–17. Magnetic resonance imaging is a use ul a junctive


imaging technique or iagnosing eep-vein
thrombosis or all EXCEPT which o the ollowing
reasons?
a. Lower cost than compression ultrasonography
b. Improve evaluation o the ilio emoral veins
c. Ability to iagnosis nonthrombotic con itions
d. Ability to reconstruct pelvic venous system
anatomy

52–18. d-Dimer concentrations can be elevate in which o


the ollowing pregnancy-relate complications?
a. Preeclampsia
b. Placenta previa
c. Gestational iabetes
d. Prurigo o pregnancy
Thromboembolic Disorders 347

52–21. Pulmonary embolism occurs in approximately what 52–26. What is the most common presenting symptom in
percentage o patients with untreate eep-vein patients with a pulmonary embolus?
thrombosis? a. Cough
a. 10% b. Dyspnea
b. 35% c. Syncope
c. 60% d. Chest pain
d. 85%
52–27. Results rom which o the ollowing tests, when
52–22. I un ractionate heparin is selecte as the treatment normal, e ectively exclu e a iagnosis o pulmonary
or thromboembolism, the initial intravenous ose embolism?
is continue or at least how many ays be ore a. Chest ra iograph
converting to subcutaneous osing? b. Electrocar iogram
a. 2 c. Oxygen saturation
b. 5 d. None o the above
c. 10
d. 14 52–28. Circulatory collapse requires obstruction o at least
what percentage o the pulmonary vascular tree?
52–23. In women who are ully anticoagulate with low- a. 25%
molecular-weight heparin, perio ic monitoring o b. 50%
anti- actor Xa levels is particularly important when
which o the ollowing comorbi ities is present? c. 75%
a. Pneumonia d. 90%
b. Renal insu iciency
52–29. Which o the ollowing clinical in ings in the
c. Chronic hypertension setting o pulmonary embolism increases the
d. Gestational iabetes mortality rate?
a. Hypoxia
52–24. Women receiving therapeutic oses o low- b. Hemoptysis
molecular-weight heparin shoul not receive
c. Marke elevation o d- imers
neuraxial blocka e (e.g., epi ural anesthesia) or how
long a ter the last ose was a ministere ? d. Right ventricular ys unction
a. 12 hours
b. 24 hours 52–30. What is the proce ure-relate mortality rate with
pulmonary angiography?
c. 36 hours
a. 0.005%
d. 48 hours
b. 0.05%
52–25. he risk or heparin-in uce osteoporosis is c. 0.5%
increase by all EXCEPT which o the ollowing? d. 5%
a. Cigarette smoking
b. A rican American race
c. Anticoagulation longer than 6 months
d. Un ractionate heparin as the anticoagulant
348 Medical and Surgica l Complications

52–31. A35-year-ol G4P3 with three prior cesarean 52–32. Which o the ollowing statements is accurate
eliveries presents at 39 weeks’gestation regar ing the use o thrombolytics in pregnancy?
complaining o rupture etal membranes an a. hey shoul be avoi e entirely.
shortness o breath. During preoperative evaluation, b. hey shoul be avoi e in the last trimester.
she is note to have tachycar ia an hypoxia,
c. hey shoul be avoi e in the irst trimester.
an compute tomographic (C ) angiography is
emergently per orme . One image rom this stu y is d. hey shoul be a ministere when clinically
shown here, an the arrow points to a illing e ect. in icate irrespective o pregnancy timing.
Which o the ollowing is the most appropriate next
step?

Repro uce with permission rom Cunningham FG, Leveno KJ, Bloom SL, et al (e s): T
romboembolic isor ers. In Williams Obstetrics, 24th e . New York, McGraw-Hill, 2014,
Figure 38-5.

a. Pulmonary angiography
b. hrombolytic a ministration
c. Placement o a vena cava ilter
d. Un ractionate heparin in usion
366

Hematological Disorders

56–1. A hemoglobin concentration below which of the 56–4. For the patient described in Question 56–3, what is
following thresholds would indicate anemia in the most appropriate initial treatment?
an iron-supplemented pregnant woman inany a. Red cell transfusion
trimester? b. Folic acid, 4 mg orally daily
a. 9.0 g/dL c. Hydroxyurea, 1 g orally daily
b. 9.5 g/dL d. Elemental iron, 200 mg orally daily
c. 10.0 g/dL
d. 10.5 g/dL 56–5. Of medical conditions associated with anemia
of chronic disease, which is most frequently
56–2. What is the most common cause of antepartum encountered in pregnancy?
anemia in pregnant women? a. Crohn disease
a. Thalassemia b. Hodgkin lymphoma
b. Iron deficiency c. Chronic renal insufficiency
c. Folic acid deficiency d. Systemic lupus erythematosus
d. Anemia of chronic disease
56–6. A 36-year-old G3P2 at 18 weeks’ gestation reports
56–3. A 19-year-old primigravida at 29 weeks’ gestation extreme fatigue and is found to be have a hemoglobin
is noted to have anemia with a hemoglobin concentration of 7.5 g/dL. The erythrocyte mean
concentration of 8 g/dL. The peripheral blood smear corpuscular volume is markedly elevated and measures
below is obtained. Which of the following laboratory 124 fL. A peripheral blood smear is obtained and is
findings are likely to accompany this condition? shown here. What is the most likelyetiology?

Reproduced with permission from Beutler E: Disorders of iron metabolism. In Lichtman


Reproduced with permission from Green R: Folate, cobalamin, and megaloblastic anemia.
MA, Kipps TJ, Seligsohn U (eds): Williams Hematology, 8th ed. New York, McGraw-Hill,
In Lichtman MA, Kipps TJ, Seligsohn U (eds): Williams Hematology, 8th ed. New York,
2010, Figure 42-10C.
McGraw-Hill, 2010, Figure 41-12A.

a. Decreased serum ferritin level a. Iron deficiency


b. Elevated mean corpuscular volume b. Vitamin B6 deficiency
c. Decreased total iron binding capacity c. Folic acid deficiency
d. Positive sickle-cell screen (Sickledex) result d. Vitamin B12 deficiency
Hema tological Disorders 367

56–7. All EXCEPT which of the following statements 56–11. Which of the following statements is true regarding
regarding autoimmune hemolytic anemia in women who are heterozygous for the glucose-6-
pregnancy are true? phosphate dehydrogenase mutation?
a. Pregnancy can acceleratehemolysis. a. Infections in pregnancy can precipitatehemolysis.
b. The direct Coombs test is usually positive. b. Some degree of protection against malaria is
c. The indirect Coombs test is usually positive. conferred.
d. The cause of aberrant antibody production c. Lyonization results in a variable degree of enzyme
originates from fetal microchimerism. activity.
d. All of the above
56–8. Pregnant women with paroxysmal nocturnal
hemoglobinuria are at increased risk for which of the 56–12. Which of the following pregnancy complications has
following? been associated with polycythemia vera?
a. Renal failure a. Stillbirth
b. Venous thrombosis b. Coagulopathy
c. Maternal mortality c. Placenta previa
d. All of the above d. Placental abruption

56–9. What is the typical inheritance pattern of the 56–13. What is the prevalence of sickle-cell trait among
mutation in the spectrin gene that results in African-American women in the United States?
hereditary spherocytosis? a. 1%
a. Mitochondrial b. 4%
b. X-linked dominant c. 8%
c. Autosomal dominant d. 25%
d. Autosomal recessive
56–14. In patients with sickle-cell disease, red cells may
56–10. The following image demonstrates the appearance assume the following configuration under which of
of spherocytes using scanning electron microscopy. the following conditions?
In addition to identifying erythrocytes with this
appearance, what other laboratory finding helps
confirm the diagnosis of hereditary spherocytosis? a. Hyperglycemia
b. Low oxygen tension
a. Low serum haptoglobin level c. Dietary protein deficiency
b. Elevated serum bilirubin level d. Administration of certain antibiotics
c. Increased erythrocyte osmotic fragility
d. Increased serum level of fibrin split products
368 Medical and Surgica l Complications

56–15. A 22-year-old primigravida with sickle-cell disease 56–18. Which of the following contraceptive choices may
presents with complaints of fever, cough, and help prevent painful crises in women with sickle-cell
increasing dyspnea. The following chest radiograph disease?
is obtained. All EXCEPT which of the following are a. Intrauterine device
precipitants of this condition? b. Surgical sterilization
a. Infection c. Depot medroxyprogesterone acetate
b. Atelectasis d. Combination oral contraceptive pills
c. Coagulopathy
d. Marrow emboli 56–19. Sickle-cell trait (hemoglobin AS) has been associated
with an increased risk for which of the following?
a. Preeclampsia
b. Placental abruption
c. Gestational diabetes
d. Urinary tract infections

56–20. Women with which of the following α-globin


genotypes are at risk to have offspring with
homozygous α -thalassemia?
a. αα /--
b. α -/αα
c. α -/α -
d. αα / αα

56–21. Which of the following findings on hemoglobin


electrophoresis would be most consistent with a
diagnosis of β -thalassemia minor?
a. Hemoglobin A2 less than 1%; normal fetal
hemoglobin level
b. Hemoglobin A2 less than 1%; fetal hemoglobin
greater than 2%
56–16. Pregnant women with sickle-cell anemia are at c. Hemoglobin A2 greater than 3.5%; normal fetal
increased risk for which of the following maternal hemoglobin level
complications?
d. Hemoglobin A2 greater than 3.5%; fetal
a. Renal failure hemoglobin greater than 2%
b. Cardiomyopathy
c. Pulmonary hypertension 56–22. What is the most common cause of
d. All of the above thrombocytopenia in pregnancy?
a. Severe preeclampsia
56–17. Which of the following statements is accurate b. Consumptive coagulopathy
regarding the use of prophylactic red cell transfusions c. Gestational thrombocytopenia
for pregnant women with sickle-cell anemia? d. Immune thrombocytopenic purpura
a. Gestation duration is increased.
b. Perinatal mortality rate is decreased. 56–23. When indicated in pregnancy, which of the
c. Rate of fetal-growth restriction is decreased. following is the most appropriate initial treatment of
d. Risk of red cell alloimmunization is increased. immune thrombocytopenic purpura?
a. Azathioprine
b. Laparoscopic splenectomy
c. Systemic corticosteroids
d. Intravenous anti-D immunoglobulinG
Hema tological Disorders 369

56–24. Which of the following management strategies is 56–29. Hemophilia A, characterized by a severe deficiency
recommended to detect fetal thrombocytopenia in factor VIII, is inherited in what fashion?
in women who have chronic immune a. X-linked dominant
thrombocytopenic purpura? b. X-linked recessive
a. Cordocentesis c. Autosomal dominant
b. Scalp sampling d. Autosomal recessive
c. Cesarean delivery
d. None of the above 56–30. In women affected by hemophilia A, the risk for
excessive hemorrhage at delivery can be reduced by
56–25. Most cases of thrombocytosis in which the platelet all EXCEPT which of the following?
count exceeds 1 million/µL are caused by which of a. Uterotonics
the following conditions? b. Desmopressin
a. Malignancy c. Avoiding episiotomy
b. Iron deficiency d. Operative vaginal delivery
c. Autoimmune conditions
d. Essential thrombocytosis 56–31. Pregnancy physiology results in which of the
following changes to factor VIII and von Willebrand
56–26. Although there is considerable clinical overlap factor (vWF) levels?
between thrombotic thrombocytopenic purpura a. Increased factor VIII and vWF levels
(TTP) and hemolytic uremic syndrome (HUS),
b. Decreased factor VIII and vWF levels
these entities can be distinguished by all EXCEPT
which of the following? c. Increased factor VIII levels; decreased vWF factor
levels
a. HUS has more renal dysfunction.
d. Decreased factor VIII levels; increased vWF factor
b. HUS is seen primarily in adults. levels
c. TTP more frequently has associated neurologic
aberrations. 56–32. Although pregnancy outcomes are generally good in
d. All statements are true. women who have von Willebrand disease, which of
the following pregnancy-related complications may
56–27. What is the cornerstone of treatment for thrombotic be encountered in up to 50 percent of such cases?
thrombocytopenic purpura? a. Preterm birth
a. Anticoagulation b. Placental abruption
b. Plasmapheresis c. Postpartum hemorrhage
c. Platelet transfusion d. Fetal-growth restriction
d. Intravenous immunoglobulin (IVIG)

56–28. Because the treatment of the two conditions is very


different, differentiating severe preeclampsia from
thrombotic thrombocytopenic purpura (TTP) is
important. Which of the following clinical findings
favors TTP?
a. Moderate thrombocytopenia
b. Severe hemolysis
c. Mild disseminated intravascular coagulation
d. Marked transaminase elevation
371

CHAPTER00

Diabetes Mellitus

57–1. The number of Americans with diabetes is


increasing, in part, because of which of the
following?
a. An obesity prevalence that has plateaued
b. An aging population that is more likely to
develop type 2 diabetes
c. Decreased populations within minority groups,
who are more likely to develop type 1 diabetes
d. All of the above

57–2. In 2006 in the United States, the rate of diabetes


during pregnancy approximated which of the
following?
a. 0.5%
b. 4%
c. 9% 57–3. The graphic below concerning the age-specific
d. 14% incidence of gestational diabetes suggests which of
the following?

Reproduced with permission from Cunningham FG, Leveno KJ, Bloom SL, et al (eds):
Diabetes mellitus. In Williams Obstetrics, 24th ed. New York, McGraw-Hill, 2014,
Figure 57-1.

a. The greatest increase in incidence was in the


group of women ≥ 35 years.
b. For all maternal age groups, the incidence of
gestational diabetes has increased since 1989.
c. The decreases seen in the group of women
younger than 25 years may be explained by
improved dietary intake.
d. None of the above

57–4. A 27-year-old woman with proliferative retinopathy


would have what diagnosis by the Whiteclassification?
a. R diabetes
b. H diabetes
c. RF diabetes
d. A2 gestationaldiabetes
372 Medical and Surgica l Complications

57–5. What fasting plasma glucose level is used as the 57–9. Fetal hyperinsulinemia in the second half of
threshold to diagnose overt diabetes? pregnancy is associated with which of the following?
a. 105 mg/dL a. Altered fetal growth
b. 116 mg/dL b. Neonatal hypoglycemia
c. 126 mg/dL c. Maternal hyperglycemia
d. 140 mg/dL d. All of the above

57–6. Which of the following is a risk factor in pregnant 57–10. What might be said of the pregnancy yielding this
women for impaired carbohydrate metabolism? 6050 g newborn?
a. Family history of diabetes
b. Previous infant with polycystic kidney disease
c. High serum levels of antiphospholipid antibodies a. The baby was at risk for neonatalhyperglycemia.
d. All of the above b. The mother probably had excellent glycemic
control.
57–7. Fetuses of overtly diabetic mothers have an increased c. The mother had an increased risk for shoulder
risk for which of the following? dystocia.
a. Preterm delivery d. All of the above
b. Spontaneous abortion
c. Congenital malformation
d. All of the above

57–8. This figure illustrates the frequency of congenital


malformations at given maternal glycohemoglobin
levels early in pregnancy. What can be said regarding
this relationship?

a. Gestational diabetes confers an increased risk of


congenital malformation.
b. If the glycohemoglobin A1C is < 6, there is no
risk of congenital malformation.
c. As preconceptional glucose control worsens, the
incidence of congenital malformation increases.
d. The highest risk for congenital malformation is
seen with a glycohemoglobin A1C level between 7
and 8.
Dia betes Mellitus 373

57–11. This graphic comparing birthweight distributions of 57–14. What is the most likely cause for the increased
neonates born to diabetic and nondiabetic mothers incidence of respiratory distress syndrome in the
illustrates which of the following? neonates of diabetic mothers?
a. Indicated preterm delivery
27 700
b. Delayed maturation of type II pneumocytes
a. Newborns of diabetic mothers are“growth c. Decreased production of surfactant in a
promoted.” hyperglycemic environment
b. The birthweight distribution of newborns of d. All of the above
diabetic mothers is skewed toward consistently
heavier birthweights. 57–15. Hyperbilirubinemia in the newborns of diabetic
c. Growth promotion from maternal hyperglycemia mothers is related to which of the following?
does not precluded fetal-growth restriction, which a. Newborn polycythemia
is defined as 2 standard deviations below the
b. Relative fetal hypoxia
mean.
d. All of the above c. Hyperglycemia-mediated increases in maternal
affinity for oxygen and fetal oxygen consumption
d. All of the above

57–12. The incidence of stillbirth is highest in pregnancies 57–16. What can be said regarding the inheritance of
complicated by which of the following? diabetes?
a. Overt diabetes a. Breast feeding increases the inheritance risk.
b. Gestational diabetes b. There is a genetic component to the inheritance
c. Overt diabetes and hypertension of type I diabetes.
d. Gestational diabetes and hypertension c. If both parents have type 2 diabetes, the risk of
inheritance approximates 20%.
57–13. Which of the following is a reasonable explanation d. None of the above
for hydramnios in diabetic pregnancy?
a. Maternal endothelial leak caused by 57–17. What is the maternal mortality rate associated with
hyperglycemia class H diabetes?
b. Glucose reabsorption by the fetal glomerular a. 10%
collecting system b. 25%
c. Osmotic gradient created by high glucose c. 50%
concentrations in the amnionic fluid
d. 75%
d. All of the above
57–18. For women with chronic hypertension and diabetic
nephropathy, what is the associated risk of
developing preeclampsia?
a. 20%
b. 40%
c. 60%
d. 80%

57–19. The adverse effects of pregnancy on diabetic


retinopathy may be reduced by which of the
following during pregnancy?
a. Good glycemic control
b. Laser retina photocoagulation in pregnancy
c. Folate, 4 mg orally daily throughout pregnancy
d. A and B
374 Medical and Surgica l Complications

57–20. What is the most important component of diabetic 57–26. Concerning labor and delivery in insulin-requiring
ketoacidosis treatment in pregnancy? diabetics, which of the following is true?
a. Restore euglycemia a. The patient should receive insulin as usual and
b. Provide intravenous hydration have breakfast prior to induction.
c. Provide intravenous potassium repletion b. The mother should be adequately hydrated, and
d. Provide intravenous bicarbonate to correct euglycemia is maintained by a continuous insulin
acidosis infusion.
c. Primary cesarean is preferred for any insulin-
57–21. Which of the following infections is increased in requiring diabetic mother whose baby has an
estimated fetal weight ≥ 4000 g.
gravidas with overt diabetes?
d. Euglycemia is best achieved by administering
a. Pyelonephritis
dextrose-containing intravenous fluids and
b. Respiratory infections regularly scheduled insulin injections.
c. Wound infection after cesarean delivery
d. All of the above 57–27. Which of the following defines gestational diabetes?
a. Any diabetes that is first detected in pregnancy.
57–22. Concerning the preconceptional period, what can be b. Diabetes that does not require insulin during
said of care for the diabeticwoman? pregnancy.
a. Should achieve euglycemia c. A glycohemoglobin A1C level < 7 found early in
b. Should begin dailyfolate pregnancy.
c. Should have an ophthalmological appointment to d. None of the above
screen for retinopathy
d. All of the above 57–28. Concerning the screening of gravidas for gestational
diabetes, which of the following is true?
57–23. Women with type 1 diabetes should achieve a. Approximately 80% of institutions in the United
glycemic control with which of the following during States use universal screening.
pregnancy? b. Universal screening is endorsed by the
a. Insulin International Workshop Conference on
b. Diet alone Gestational Diabetes.
c. Insulin and diet c. The American College of Obstetricians and
d. Oral hypoglycemic agents Gynecologists (ACOG) endorses one-step glucose
screening that uses a 75-g glucose tolerance test.
d. The American College of Obstetricians and
57–24. During which of the following epochs in pregnancy
Gynecologists (ACOG) endorses a two-step
is the peak incidence of maternal hypoglycemia
screening process that begins with a 50-g oral
noted?
glucose screen.
a. 10–14 weeks
b. 20–24 weeks 57–29. Reported by Landon (2009), the Maternal-Fetal
c. 28–32 weeks Medicine Units Network evaluated women with
d. 34–38 weeks mild gestational diabetes, defined as a fasting blood
glucose level > 95 mg/dL. Compared with women
receiving only standard obstetrical care, those
57–25. In pregnant women with overt diabetes, which of
receiving dietary counseling and glucose monitoring
the following statements is true regardingdetection
had decreased rates of which of the following
of fetal anomalies?
morbidities?
a. Sonographic anatomical evaluation is best a. Fetal macrosomia
performed at 18–22 weeks’gestation.
b. Cesarean delivery
b. The accuracy of sonographic fetal anatomical
c. Shoulder dystocia
evaluation is not diminished by concurrent
maternal obesity. d. All of the above
c. Maternal serum alpha-fetoprotein determination
should be completed early, between 12 and
16 weeks, in mothers with overt diabetes.
d. None of the above
Dia betes Mellitus 375

57–30. In women with gestational diabetes, early fasting 57–35. Concerning glucose monitoring in pregnancy, which
hyperglycemia is associated with increased rates of of the following is true?
which of the following? a. Fasting blood glucose levels < 100 mg/dL should
a. Fetal macrosomia be the goal.
b. Cesarean delivery b. Preprandial surveillance is superior to
c. Maternal hypertension postprandial testing.
d. All of the above c. Daily monitoring with reflectance meters may
reduce fetal macrosomia rates.
57–31. Which of the following factors have been implicated d. Weekly monitoring to achieve fasting blood
in fetal macrosomia? glucose levels < 110 mg/dL reduces cesarean
a. Leptin delivery rates.
b. C-peptide
57–36. Which of the following is appropriate initial insulin
c. Insulin-like growth factor
dosing for pregnant women who meet criteria for
d. All of the above therapy?
a. 0.5 units/kg NPH, administered daily in two
57–32. As reported by Metzger (2007), the Fifth divided doses
International Working Conference on Gestational
b. 20 units NPH and 10 units regular insulin,
Diabetes recommended fasting blood glucose levels
administered daily each morning
be kept below what value?
a. 95 mg/dL c. 0.7–1.0 units/kg insulin, using a combination of
NPH and regular insulin, administered daily in
b. 100 mg/dL divided doses
c. 110 mg/dL d. None of the above
d. 120 mg/dL
57–37. Regarding metformin therapy for gestational
57–33. Concerning nutritional instructions for women with diabetes, which of the following is true?
gestational diabetes, which of the following is true? a. It crosses the placenta.
a. Daily caloric intake should range between 30 and b. Compared with insulin therapy, rates of neonatal
35 kcal/kg. hypoglycemia are increased.
b. A carbohydrate-controlled diet should maintain c. Compared with insulin therapy, rates of adverse
mild hyperglycemia and avoid ketosis. perinatal outcomes are increased.
c. The American College of Obstetricians and d. All of the above
Gynecologists (ACOG) recommends that
carbohydrate intake should comprise no more 57–38. Your patient is a 33-year-old G3P2 white female
than 20% of the total daily calories. who presents for her first prenatal visit at 16 weeks’
d. All of the above gestation. She had a newborn weighing 9 lb and
gestational diabetes in her last pregnancy. At this
57–34. Which of the following reflects the American visit, all EXCEPT which of the following are
College of Obstetricians and Gynecologists (ACOG) appropriate prenatal laboratory tests?
recommendations concerning activity in gravidas a. Rapid plasma reagin test
with gestational diabetes? b. Serum total cholesterol level
a. Modified bed rest near term c. 50-g oral glucose tolerance test
b. Moderate exercise for those without d. Quad screen to assess fetal risk of aneuploidy and
contraindications neural-tube defect
c. Maintenance of the same exercise routine
established before conception
d. None of the above
376 Medical and Surgica l Complications

57–39. Testing of the patient in Question 57–38 shows 57–41. Ms. Smith is diagnosed with gestational diabetes that
a negative quad screen result, a glucose level of is controlled solely with diet. Which of the following
146 mg/dL, and a negative rapid plasma reagin test are important for the management of her pregnancy
result. Which of the following should be ordered at term?
next? a. Induction at 38 weeks’gestation
a. 3-hour oral glucose tolerance test b. Weekly umbilical artery Doppler studies
b. Genetic counseling and offer genetic c. Cesarean delivery if the estimated fetal weight is
amniocentesis 4250 g
c. Microhemagglutination assay Treponema pallidum d. None of the above
(MHA-TP)
d. None of the above
References
57–40. Three-hour oral glucose tolerance testing of the HAPO Study Cooperative Research Group: Hyperglycemia and
patient in Question 57–38 yields normal results. adverse pregnancy outcomes. N Engl J Med 358:2061, 2008
Subsequently, she returns to her prenatal clinic at Landon MB, Spong CY, Thom E, et al: A multicenter, random- ized
treatment trial of mild gestational diabetes. N Engl J Med
normal intervals. At 26 weeks’ gestation, what test
361(14):1339, 2009
should be ordered?
Metzger BE, Buchanan TA, Coustan DR, et al: Summary and recom-
a. Rapid plasma reagin test mendations of the Fifth International Workshop-Conference on
b. Targeted fetal sonography Gestational Diabetes. Diabetes Care 30(Suppl 2):S251, 2007
c. 3-hour glucose tolerance test
d. 50-g oral glucose tolerance screen
378

Endocrine Disorders

58–1. Which of the following is true of thyroid-stimulating 58–5. The condition shown here is commonly associated
hormone (TSH) during pregnancy? with which of the following maternal signs?
a. Decreased levels are found in early pregnancy.
b. TSH crosses the placenta and stimulates fetal
thyroxine production. a. Tachycardia
c. TSH levels are increased in early pregnancy b. Enophthalmos
because of the effects of human chorionic c. Maternal androgen insensitivity
gonadotropin (hCG). d. All of the above
d. None of the above

58–2. Maternal sources account for what percentage of


fetal thyroxine at term?
a. 10%
b. 30%
c. 50%
d. 70%

58–3. In a study of more than 1000 women with thyroid


peroxidase (TPO) antibodies, there was an associated
increased risk of which of the following?
a. Placenta previa
b. Preterm delivery
c. Placental abruption
d. Preterm rupture of membranes

58–4. Which clinical symptom is not characteristic of mild


thyrotoxicosis?
a. Thyromegaly
b. Tachycardia
c. Cold intolerance
d. Poor maternal weight
Endocrine Disorders 379

58–6. The following fetal condition is associated with fetal 58–9. Which of the following is the initial treatment
thyrotoxicosis. If identified antenatally, the mother consideration for thyrotoxicosis in the pregnant
may be treated with which of the following agents, woman?
which is subsequently transported transplacentally a. Administer sodium iodide
to the fetus? b. Initiate corticosteroids
c. Administer potassium iodide
a. Iodine d. None of the above
b. Prednisone
c. Propylthiouracil (PTU) 58–10. Untreated maternal thyrotoxicosis may lead to which
d. Intravenous immune globulin G (IVIG) of the following fetal complications?
a. Stillbirth
b. Macrosomia
c. Postterm gestation
d. All of the above

58–11. Subclinical hyperthyroidism is characterized by which


of the following changes in serum hormone levels?
a. High total thyroxine level
b. Low free thyroxine (FT4) level
c. Low thyroid-stimulating hormone and normal
FT4 levels
d. High thyroid-stimulating hormone and normal
FT4 levels

58–12. Which of the following is true of routine thyroid-


stimulating hormone screening in pregnancy?
a. It is advocated by the American Association of
Clinical Endocrinologists.
b. It is not advocated by the American College of
Obstetricians and Gynecologists (ACOG).
58–7. If a pregnant woman develops fever or sore throat c. It leads to improved outcome in neonates whose
while taking a thionamide, how should this be mothers have subclinical hypothyroidism.
managed? d. All of the above
a. Stop the medication immediately
b. Increase her dose of propylthiouracil 58–13. Severe hypothyroidism is uncommon in later
pregnancy for which of the following reasons?
c. Change from propylthiouracil to methimazole
a. It is often associated with infertility.
d. None of the above
b. It is associated with increased abortion rates.
c. Most women with severe hypothyroidism elect
58–8. Which of the following is an appropriate initial first-trimester termination.
thionamide dose for pregnant women with
hyperthyroidism and is the one typically used at d. A and B
Parkland Hospital?
a. 50 mg po tid 58–14. Concerning initial treatment of maternal
hypothyroidism, which of the following is true?
b. 100 mg po tid
c. 200 mg po tid a. Approximately 400 µg thyroxine should be given
orally daily.
d. 300 mg po bid
b. Thyroid-stimulating hormone (TSH) levels
should be measured at 2-week intervals.
c. The thyroxine dose should be adjusted in 25- to
50-µg increments to achieve TSH levels between
0.5 and 2.5 mU/L.
d. All of the above
380 Medical and Surgica l Complications

58–15. An increased incidence of which of the following 58–21. Which of the following is true of thyroid nodules
pregnancy outcomes have been linked with maternal during pregnancy.
subclinical hypothyroidism? a. Are poorly assessed by fine-needle aspiration
a. Stillbirth b. Can be safely removed before 24 weeks’ gestation
b. Preeclampsia c. When smaller than 0.5 cm, can be reliably
c. Placenta previa detected sonographically
d. None of the above d. If cancerous, confer a worse prognosis than if
found in nonpregnant controls
58–16. What condition is described by a low serum free
thyroxine level and normal thyroid-stimulating 58–22. Which of the following is true regarding parathyroid
hormone values? hormone activity in pregnancy?
a. Isolated hypothyroxinemia a. It acts directly on the bone and kidney.
b. Isolated hyperthyroxinemia b. It maintains intracellular calcium concentration.
c. Subclinical hypothyroidism c. It acts indirectly on the small intestine through its
d. Subclinical hyperthyroidism effects on vitamin D synthesis and lowers serum
calcium levels.
58–17. Mild iodine deficiency during pregnancy typically d. None of the above
may cause which of the following?
a. Neonatal goiter 58–23. What is true concerning parathyroid hormone
(PTH) during pregnancy?
b. Endemic cretinism
a. Maternal PTH levels are increased.
c. Placental abruption
d. Neurodevelopmental abnormalities in offspring b. Maternal PTH leads to total serum calcium levels
that are higher than nonpregnant levels.
c. PTH that provides the greatest clinical effects is
58–18. What is the incidence of congenital hypothyroidism?
probably of placental and decidual origin.
a. 1/500 d. All of the above
b. 1/1000
c. 1/3000 58–24. Which of the following is true of
d. 1/9000 hyperparathyroidism?
a. It is generally a disease of young females.
58–19. Which of the following is true of postpartum b. It has a reported prevalence of 2–3 per 10,000
thyroiditis? women.
a. It affects 5–10% of women during the first year c. It is caused mainly by hyperfunctioning of all
postpartum. four parathyroid glands.
b. It develops at a higher rate in women with type 1 d. It may be masked by pregnancy due to significant
diabetes mellitus. calcium shunting to the fetus.
c. It is related to increasing serum levels of thyroid
autoantibodies. 58–25. Which management strategies are considered
d. All of the above appropriate for the pregnant patient with
hyperparathyroidism?
58–20. In regard to postpartum thyroiditis, which of the a. With hypercalcemic crisis, intravenous saline
following is true? infusion and diuresis should be implemented.
a. Initially, there is a hypothyroid phase. b. If a gravida is symptomatic, then oral calcium,
b. Initial hyperthyroidism responds well to 1–1.5 g daily in divided doses, is implemented.
thionamides. c. If a gravida is asymptomatic, surgical removal
c. Beta blockers may be of help in the initial phase should be performed early and before the end of
of postpartum thyroiditis. the first trimester.
d. All of the above d. Hyperkalemia and hypermagnesemia are
complications of hypercalcemic crisis, and levels
of these electrolytes should be monitored.
Endocrine Disorders 381

58–26. What is true concerning hypocalcemia in pregnancy? 58–29. Which of the following is preferred medical
a. The most common cause is renal insufficiency. therapy in pregnancy for hypertension with
b. Maternal treatment consists of large oral doses of pheochromocytoma?
phosphate. a. Hydralazine
c. Associated hypoparathyroidism follows 20% of b. Beta blockers
thyroidectomy cases. c. Alpha blockers
d. Hypocalcemic gravidas may have neonates with d. Calcium-channel blockers
bone demineralization.
58–30. Concerning Cushing syndrome in general, which of
58–27. Which of the following is true of a the following is true?
pheochromocytoma? a. Most cases arise from long-term corticosteroid
a. It is called the 10-percenttumor. treatment.
b. It is found in 0.1% of hypertensive patients b. This syndrome is rare, with an annual incidence
during pregnancy. of 1 in 3000.
c. It is detected by a 24-hour urine collection c. Occasionally, severe Cushing-associated estrogen
for free catecholamines, metanephrines, or excess may lead to severe feminization.
vanillylmandelic acid (VMA).
d. All of the above
d. All of the above
58–31. Complications of Cushing syndrome in pregnancy
58–28. The patient whose magnetic resonance (MR) include which of the following?
imaging is shown here has a history of hypertension,
a. Greater than 50% of mothers will have
palpitations, and frequent flushing episodes. The
hypertension.
most likely diagnosis is which of the following?
b. Greater than 50% of mothers will develop
diabetes mellitus.
a. Wilms tumor c. Maternal Cushing syndrome carries a maternal
b. Adrenal tumor mortality rate of 7%.
c. Liver hepatoma d. All of the above
d. Renal medulloblastoma
58–32. Concerning Addison disease, which of the following
is true?
a. The most frequent cause istuberculosis.
b. Cortisone therapy may be discontinued
postpartum.
c. Adrenal hypofunction does not usually affect
fertility.
d. Low serum cortisol levels in pregnancy should
prompt adrenocorticotropic hormone (ACTH)
stimulation testing.

58–33. Hyperaldosteronism is pregnancy is most likely


caused by which of the following?
a. Adrenal carcinoma
b. Adrenal aldosteronoma
c. Bilateral adrenal hyperplasia
d. None of the above

58–34. Normal prolactin levels in pregnancy are considered


those below what threshold?
a. 12 pg/mL
b. 18 pg/mL
c. 25 pg/mL
d. 30 pg/mL
383

CHAPTER00

Connective-Tissue Disorders

59–1. All EXCEPT which of the following isor ers are 59–4. A patient presents with photosensitivity an the
acquire connective tissue isor ers? rash seen here. What is the best screening test for
a. Rheumatoi arthritis systemic lupus erythematosus?
b. Osteogenesis imperfecta
c. Systemic lupus erythematosus
Use with permission from Dr. Martha Rac.
d. Antiphospholipi antibo y syn rome
a. Anti-Smith antibo y
59–2. Which of the following statements is true regar ing
immune-me iate connective tissue isor ers? b. Antinuclear antibo y
c. Antiphospholipi antibo y
a. hey typically have no renal involvement.
d. Anti- ouble-stran e -DNA antibo y
b. hey have a clearly eluci ate pathogenesis.
c. hey are always associate with rheumatoi 59–5. How early can fetal cells an free fetal DNA be
factor. etecte in maternal bloo ?
d. hey may or may not have an association with a. 1st trimester
autoantibo y formation. b. 2n trimester
c. 3r trimester
59–3. How shoul a patient with a connective-tissue d. Puerperium
isor er be counsele regar ing isease activity
uring pregnancy? he “autoimmunity gene,” which pre isposes to
a. It will worsen. 59–6.
lupus, rheumatoi arthritis, an Crohn isease, is
b. It will improve. locate on what chromosome?
c. It will be unchange . a. Chromosome 6
d. It will be mo ulate by the effect of pregnancy b. Chromosome 16
hormones. c. Chromosome 21
d. Chromosome 22
384 Medical and Surgica l Complications

59–7. Which systemic lupus erythematosus-specific 59–13. Base on the 1997 Revise Criteria of the American
antibo y correlates with nephritis an vasculitis Rheumatism Association, a patient coul be
activity when seen in high titers? iagnose with systemic lupus erythematosus if she
a. Anti-Ro ha which of the following fin ings?
b. Anti-La a. Diarrhea, arthritis, anemia, weight loss
c. Antinuclear b. Malar rash, anemia, oral ulcers, anti-Smith
d. Anti- ouble-stran e -DNA antibo ies
c. Discoi rash, renal failure, increase antinuclear
59–8. Which autoantibo y is associate with thrombosis, antibo y titers
fetal loss, an thrombocytopenia? d. History of preeclampsia, antiphospholipi
a. Anti-Ro antibo ies, fetal loss
b. Antinuclear
59–14. What is the inci ence of systemic lupus
c. Antiplatelet
erythematosus in pregnancy?
d. Antiphospholipi
a. 1:1200
b. 1:3300
59–9. Ninety-five percent of patients with systemic lupus
erythematosus experience all EXCEPT which of the c. 1:6000
following clinical manifestations? d. 1:10,000
a. Fever
b. Arthralgias 59–15. A patient presents for preconceptional counseling
with a history of systemic lupus erythematosus
c. Proteinuria
(SLE). She was iagnose as an a olescent but has
d. Weight loss one well uring the past few years an is currently
in remission. She wants to know what her chances
59–10. Which of the following autoantibo ies are specific are of having an uncomplicate pregnancy. Which
for lupus? of the following oes not factor into the obstetrical
a. Anti- ouble-stran e -DNAan anti-Ro outcome of a patient with SLE?
b. Anti- ouble-stran e -DNAan anti-La a. ime since last flare
c. Anti-Smith an anti-RNP b. Presence of antinuclear antibo ies
d. Anti-Smith an Anti- ouble-stran e -DNA c. Presence of antiphospholipi antibo ies
d. Disease activity at the beginning of pregnancy
59–11. In a ition to systemic lupus erythematosus (SLE),
what other con itions coul present with low titers 59–16. Following counseling, the patient in Question 59–15
of antinuclear antibo y? woul like to pursue pregnancy. What is the most
a. Chronic inflammation common complication that she will likely encounter
b. Acute viral infection uring her pregnancy?
c. Autoimmune isor er other than SLE a. Anemia
b. Preeclampsia
d. All of the above
c. Deep-vein thrombosis
59–12. All EXCEPT which of the following laboratory d. Fetal-growth restriction
fin ings may be consistent with a iagnosis of
systemic lupus erythematosus? 59–17. A patient with systemic lupus erythematosus presents
a. Anemia for prenatal care. She is currently asymptomatic on
b. Leukopenia azathioprine. Her 24-hour urine collection shows 5 g
of protein. What can be sai regar ing this patient’s
c. Decrease d- imer levels potential pregnancy outcome?
d. False-positive Venereal Disease Research a. Her fetus has an increase risk of fetal eath.
Laboratory (VDRL) test result
b. Her fetus has a perinatal mortality risk > 50%.
c. Immunosuppressive treatment shoul be
iscontinue uring pregnancy.
d. She will have a poor outcome whether her isease
remains in remission or not.
Connective-Tissue Disorders 385

59–18. he patient in Question 59–17 woul like to 59–21. he chil ren of patients with systemic lupus
know how you will iagnose a pen ing lupus erythematosus may have increase rates of which of
flare. She states that her rheumatologist follows the following complications?
her complement levels routinely. What can besai a. Neonatal lupus
regar ing lupus isease activity in pregnancy an
b. Learning isor ers
serial evaluation of complement levels?
c. Congenital heart block
a. Disease activity oes not correlate well with
complement levels. d. All of the above
b. Disease activity correlates best with ecreasing
levels of C3 complement.
59–22. A postpartum patient with systemic lupus
erythematosus (SLE) is concerne after her baby was
c. Disease activity correlates best with ecreasing born with a rash. Her pe iatrician says it is relate
levels of C4 complement. to lupus. What can you tell the patient regar ing the
d. Disease activity correlates best with ecreasing cutaneous manifestations of neonatal lupus?
levels of CH50 complement. a. hey are usually transient.
b. hey never present beyon the first week oflife.
59–19. Which of the following rugs shoul be avoi e if
at all possible uring the treatment of obstetrical c. he recurrence risk in a future pregnancy
patients with systemic lupus erythematosus? approximates 5%.
a. Aspirin d. All of the above
b. Azathioprine
c. Corticosteroi s 59–23. A patient with systemic lupus erythematosus has
anti-SS-A antibo ies. She is worrie about the risk of
d. Mycophenolate mofetil the con ition seen in this M-mo e image. What is
the risk of this complication in her fetus or neonate?
59–20. Which rug is most helpful in managing a. < 1%
ermatological manifestations of systemic lupus
b. 2–3%
erythematosus?
a. Gol salts c. 6–7%
b. Azathioprine d. 10–15%
c. Cyclophosphami e
d. Hy roxychloroquine
386 Medical and Surgica l Complications

59–24. Which of the following are safe an effective 59–30. Once clinical criteria are present, all EXCEPT
contraceptive metho s for patients with systemic which of the following tests are useful in iagnosing
lupus erythematosus who lack antiphospholipi antiphospholipi antibo y syn rome?
antibo ies? a. Lupus anticoagulant
a. Progesterone onlypills b. Antiplatelet antibo ies
b. Progesterone intrauterine evice c. Anticar iolipin antibo ies
c. Combination estrogen-progesterone pills d. Anti-β 2 glycoprotein I antibo ies
d. All of the above
59–31. Which is the main mechanism by which
59–25. All EXCEPT which of the following are common antiphospholipi antibo ies cause amage?
features of antiphospholipi antibo y syn rome? a. Deactivation of the tissue factor pathway
a. hrombocytosis b. Increase protein C an protein S activity
b. Central nervous system involvement c. Increase eci ual pro uction of prostaglan in E2
c. Recurrent arterial or venous thrombosis d. Exposure of the basement membrane of
d. Fetal loss in the secon half of pregnancy en othelium an syncytiotrophoblast

59–26. Which of the following might be associate with the 59–32. A pregnant patient has known antiphospholipi
presence of anti-β 2 glycoprotein I antibo ies? antibo ies. Last year, she was hospitalize for
a. Neonatal lupus pulmonary embolism. Which of the following
b. Intervillous space thrombosis therapies is most effective uring her current
pregnancy?
c. Systemic lupus erythematosus flare
a. Glucocorticoi s
d. All of the above
b. Low- oseaspirin
59–27. Nonspecific antiphospholipi antibo ies are present c. Low-molecular-weight heparin
in low titers in what percentage of the normal d. Low- oseaspirin an unfractionate heparin
nonpregnant population?
a.1%
b.5%
c. 10%
d. 15%

59–28. Nonspecific antiphospholipi antibo ies are present


in low titers in what percentage of the normal
pregnant population?
a.1%
b.5%
c. 10%
d. 15%

59–29. Which of the following tests is least specific for lupus


anticoagulant?
a. Partial thromboplastin time
b. Dilute Russell viper venom time
c. Platelet neutralization proce ure
d. All of the above are highly specific
Connective-Tissue Disorders 387

59–33. Which statement is false regar ing the connective- 59–36. Which connective-tissue isor er is characterize by
tissue isor er in which these joint fin ings are most a necrotizing granulomatous vasculitis affecting the
classically seen? respiratory tract an ki neys?
a. Systemicsclerosis
b. akayasu arteritis
Repro uce with permission from Shah A, St. Clair W: Rheumatoi arthritis. In Longo DL,
Fauci AS, Kasper DL, et al (e s): Harrison’s Principles of Internal Me icine, 18th e . New
c. Ehlers-Danlos syn rome
York, McGraw-Hill, 2012, Figure 321-1. d. Wegener granulomatosis

a. It is more common in women than in men. 59–37. Which isor eris characterize by chronic
b. It is characterize by chronic polyarthritis. inflammatory arteritis of the great vessels?
c. It is more likely to evelop in women who have a. Systemic sclerosis
been pregnant. b. akayasu arteritis
d. Cigarette smoking increases the risk of this c. Ehlers-Danlos syn rome
isor er. d. Wegener granulomatosis

59–38. A pregnant patient presents with a characteristic


rash, shown here, an asymmetrical muscle
59–34. Which of the following factors contributes the weakness. A muscle biopsy reveals inflammation an
highest score when etermining whether a patient muscle fiber egeneration. Which of the following is
meets criteria for a iagnosis of rheumatoi arthritis? false?
a. Symptom uration > 6 weeks
b. Involvement of two largejoints a. She most likely has polymyositis.
c. Abnormal C-reactive protein levels b. An abnormal electromyogram is expecte .
d. High-positive rheumatoi factor level c. She may have an associate malignant tumor.

59–35. All EXCEPT which of the following rugs may be d. he isease typically respon s to intravenous
use safely in early pregnancy to treat rheumatoi immune globulin (IVIG) an corticosteroi s.
arthritis?
a. A alimumab
b. Leflunomi e
59–39. Which of the following inherite isor ers has been
c. Sulfasalazine associate with spontaneous uterine rupture?
d. Cyclooxygenase-2 (COX-2) inhibitors a. Chon ro ysplasia
b. Huntington isease
c. Epi ermolysis bulla
d. Ehlers-Danlos syn rome
389

CHAPTER00

Neurological Disorders

60–1. Neurovascular disorders account for what percent of 60–6. A 28-year-old woman who is 8 weeks pregnant
maternal deaths in the United States? presents with symptoms that are consistent with
a.1% a migraine headache. Which of the following
b.5% medications used to treat migraine headaches should
be avoided?
c. 10%
a. Ibuprofen
d. 20% b. Metoprolol
c. Sumatriptan
60–2. Why is magnetic resonance imaging a preferred
modality in the diagnosis of neurovascular disorders d. Ergotamine derivatives
in pregnancy?
60–7. A young woman presents for preconceptional
a. It is the most costeffective.
counseling because of her history of seizure disorder.
b. It does not involve ionizing radiation. She currently takes phenytoin and phenobarbital,
c. It is excellent for detecting recent hemorrhage. and her last seizure was 1 year ago. What should
d. None of the above she be told about her medication use and imminent
attempt at conception?
60–3. What is the most common neurologic complaint a. An attempt should be made to reduce her
during pregnancy? medications to a single drug.
a. Seizure b. She should change to valproate to reduce the risk
b. Headache of fetal teratogenicity.
c. Leg numbness c. No change in her medication regimen is indicated
d. Hand weakness if her seizure disorder is stable.
d. Since she has been seizure free for 1 year, she
60–4. Which type of headache is most likely to be affected should withdraw from all medication use.
by pregnancy-induced hormonal changes?
a. Cluster 60–8. Which of the following is not a reason for
b. ension subtherapeutic anticonvulsant levels during
pregnancy?
c. Migraine
a. Increased gastric motility
d. Intracranial nonvascular
b. Increased nausea and vomiting
60–5. Which of the following obstetrical complications c. Increased glomerular filtration
is increased in women who experience migraine d. Induction of hepatic enzymes during pregnancy
headaches?
a. Preeclampsia
b. Preterm labor
c. Premature rupture of membranes
d. None of the above
390 Medical and Surgica l Complications

60–9. Which of the following medications is not associated 60–13. What is the most common etiology for ischemic
with an increased risk of this congenital anomaly stroke in pregnancy?
when taken in early pregnancy? a. Cocaine use
b. Hypertension
c. Saccular aneurysm
a. Valproate
d. Arteriovenous malformation
b. Phenytoin
c. Carbamazepine
60–14. A 36-year-old African American primigravida who
d. None of the above is 30 weeks pregnant had a headache earlier in the
day. Her husband brings her to the emergency
60–10. Serum levels of antiepileptic medications are department after noticing she is unable to use
unreliable in pregnancy for which of the following her right hand and arm normally. Which of the
reasons? following laboratory studies is not appropriate in her
a. Altered protein binding evaluation?
b. Increased glomerular filtration a. Serum lipids
c. Levels are not available for new medications b. Alkaline phosphatase
d. None of the above c. Antiphospholipid antibody
d. Hemoglobin electrophoresis
60–11. Increased rates of which of the following have not
contributed to the increase in stroke prevalence? 60–15. he patient in Question 60–14 presents for her
a. Obesity postpartum visit and wants to discuss risks and
management during a future pregnancy. Which of
b. Heart disease
the following statements is most accurate?
c. Smoking in women
a. Prophylactic β-blocker therapy during pregnancy
d. Diabetes mellitus
is indicated.
b. here are no firm guidelines for prophylaxis
60–12. When do pregnancy-related strokes most commonly during a future pregnancy.
occur?
c. he recurrence risk is high, and she should not
a. Postpartum
pursue another pregnancy.
b. Intrapartum
d. All women with prior stroke should be given
c. First trimester prophylactic anticoagulation.
d. Second trimester
60–16. Which of the following is not an accurate statement
regarding maternal middle cerebral artery embolism
during pregnancy?
a. May be caused by paradoxical embolism
b. Occurs more commonly in the first trimester
c. Must exclude thrombosis and hemorrhage prior
to diagnosis
d. reatment includes antiplatelet therapy during
pregnancy
Neurologica lDisorders 391

60–17. How do the morbidity and mortality rates with the 60–20. During computed tomographic (C ) angiography
following lesion compare with those of subarachnoid performed as part of an evaluation for headache, the
hemorrhage? following was found in your patient. What is the risk
of lesion rupture?

a. Increased a. 0.1%
b. Decreased b. 0.5%
c. No difference c. 1%
d. No information available d. 5%

60–21. For a pregnant patient, which of the following


statements regarding the lesion seen in Question
60–18. What is the most important management strategy 60–20 is most accurate?
used to reduce complications associated with
a. he cardinal symptom of rupture is hemiparesis.
Charcot-Bouchard aneurysms?
b. Aneurysms are most likely to bleed in the first
a. Control of systolic hypertension
trimester.
b. Appropriate prophylactic anticoagulation
c. Surgical repair is the preferred treatment for
c. Prepregnancy closure of a patent foramen ovale women remote fromterm.
d. None of the above
d. Cranial computed tomography with contrast is
the preferred imaging modality.
60–19. What is the most common cause of subarachnoid
hemorrhage? 60–22. Which of the following statements regarding
a. rauma arteriovenous malformations and pregnancy is true?
b. Cerebral venous thrombosis a. he mortality rate associated with hemorrhage is
c. Ruptured saccular aneurysm 25–50%.
d. Ruptured arteriovenous malformation b. Bleeding from this lesion is more frequent during
pregnancy.
c. hey are the most common abnormality of the
cerebrovascular system encountered during
pregnancy.
d. None of the above

60–23. Which of the following organisms is suggested to act


as an environmental trigger in the development of
multiple sclerosis?
a. Influenza B
b. Herpesvirus 6
c. Chlamydia trachomatis
d. None of the above
392 Medical and Surgica l Complications

60–24. What percent of women will develop multiple 60–27. Acute exacerbations of multiple sclerosis during
sclerosis following an episode of isolated optic pregnancy can safely be treated with which of the
neuritis? following therapies?
a. 10% a. Interferon β 1a and β1b
b. 25% b. High-dose corticosteroids
c. 50% c. Intravenous immune globulin
d. 75% d. All of the above

60–25. Identification of the lesions shown in this imaging 60–28. A 34-year-old woman who was diagnosed
study is most useful in what capacity? with multiple sclerosis 2 years ago presents for
preconceptional counseling. She currently takes
baclofen and natalizumab. What she should be told
a. o determine the likelihood of a relapse about her condition?
b. o estimate the risk of obstetric complications a. Patients with multiple sclerosis have slightly
c. o confirm the initial diagnosis of multiple poorer perinatal outcomes.
sclerosis b. Intravenous immune globulin may prevent
d. o evaluate response following treatment with relapses during the puerperium.
interferon β 1a c. Baclofen and natalizumab should both be stopped
before attempting to conceive.
60–26. For patients with multiple sclerosis, which of the d. Breast feeding has a protective effect on the
following is not associated with an increased risk of postpartum relapse rate.
relapse in the puerperium?
a. Breast feeding 60–29. Antibodies to which of the following structures are
commonly found in patients with myasthenia gravis?
b. Relapses during pregnancy
a. Nonself antigens
c. High relapse rate prior to pregnancy
b. Acetylcholine receptor
d. High multiple sclerosis disability score
c. Oligodendrocyte glycoprotein
d. Muscle-specific tyrosine kinase

60–30. A 30-year-old white woman with myasthenia


gravis who is currently taking corticosteroids and
azathioprine presents for prenatal care at 12 weeks’
gestation. What is the most appropriate treatment to
manage her disease?
a. Stop azathioprine
b. Continue her current medications
c. Recommend prophylactic thymectomy
d. Start plasmapheresis in the third trimester to
reduce neonatal effects
Neurologica lDisorders 393

60–31. he patient in Question 60–30 has a prenatal 60–33. In which trimester do exacerbations of myasthenia
sonographic examination in the third trimester. gravis most commonly occur?
What is the best explanation for the findings shown a. First
here? b. Second
c. hird
A
a. Her fetus also has a diagnosis of myasthenia d. Equal rates across all trimesters
gravis.
b. he finding is unrelated to her diagnosis of 60–34. During pregnancy, what is the most appropriate
myasthenia gravis. treatment of acute Guillain-Barré syndrome?
c. She has a higher likelihood of having developed a. Cyclosporine
gestational diabetes. b. Interferon 1β 2
d. his is a transient side effect of transplacental c. Intravenous dexamethasone
passage of maternal antibodies. d. High-dose intravenous immune globulin

60–35. How is the frequency of the lesion shown here


60–32. Which of the following medications should be altered during pregnancy?
avoided during labor and delivery in a woman with a
diagnosis of myasthenia gravis?
a. Gentamicin a. Doubled
b. Succinylcholine b. No change
B c. Quadrupled
c. Magnesium sulfate
d. All of the above d. Decreased by half

60–36. Which of the following is not a recommended


therapy for the condition shown in Question
60–35?
a. Massage
b. Prednisone
c. Valacyclovir
d. Artificial tears

60–37. Which of the following complications is increased in


women who experience Bell palsy during pregnancy?
a. Preeclampsia
b. Preterm labor
c. Premature rupture of membranes
d. None of the above
394 Medical and Surgica l Complications

60–38. Which of the following complications is not 60–40. Which of the following is not a criteria for the
increased in women with spinal cord injuries? diagnosis of idiopathic intracranial hypertension?
a. Preterm labor a. Recent weight gain > 20 lb
b. Low birthweight b. Intracranial pressure > 250 mm H 2O
c. Gestational diabetes c. Normal cerebral spinal fluid chemistries
d. Asymptomatic bacteriuria d. Normal cerebral magnetic resonance imaging

60–39. Which of the following activities does not precipitate 60–41. Which of the following is not a common therapy
autonomic dysreflexia in parturients with spinal cord used to treat the development of visual field defects
injuries? associated with idiopathic intracranial hypertension
a. Amniotomy in pregnancy?
b. Foley catheter insertion a. Furosemide
c. Epidural catheter placement b. Prednisone
d. Uterine tocodynamometer transducer positioning c. Acetazolamide
d. herapeutic lumbar puncture
396

Ps chiatric Disorders

61–1. Which of the following has been associated with 61–6. A 35-year-old primipara presents to your office for
psychiatric disorders in pregnancy? routine follow-up. The patient reports feeling very
a. Substance abuse sad, anxious, and exhausted. In addition, she has
b. Poor infant outcomes a great sense of hopelessness. Even when the baby
sleeps, she finds that she cannot get to sleep. This
c. Poor obstetrical outcomes has been going on for at least a month. The patient
d. All of the above has a history of depression for which she took
medication prior to pregnancy. She denies thoughts
61–2. A 20-year-old primipara presents for her 2-week of suicide or infanticide. Which of the following
postpartum appointment. During your conversation, statements is true?
she reports that she experienced some sadness with a. For women who discontinue treatment for
crying and insomnia when she came home from the
depression, 70% relapse.
hospital. Overall, she considers herself a fairly happy
person, and those symptoms have since resolved. She b. If this patient goes untreated, there is a 25%
mentions it only because it has never happened to chance that she will be depressed 1 year from
her before. What is the most likely diagnosis? now.
a. Postpartum blues c. Her condition could lead to insecure attachment
and later behavioral problems in the child.
b. Domestic violence
c. Postpartum psychosis d. All of the above
d. Postpartum depression 61–7. You diagnose the patient in Question 61–6 with a
major depressive disorder. You think it is severe.
61–3. What is the best management plan for the patient in Your best plan of management is which of the
Question 61–2? following?
a. Emotional support a. Psychotherapy
b. Initiate bupropion b. Antidepressant treatment and psychotherapy
c. Brief course of fluoxetine c. Treatment with a mood stabilizer such as lithium
d. Weekly therapy sessions for 4–6 months d. Hospitalization under the care of a psychiatrist
61–4. Risk factors for depression include which of the 61–8. In a woman with severe depression, which of the
following? following medications might best be tried initially?
a. History of abuse a. Lithium
b. Nicotine dependence b. Citalopram
c. Family history of depression c. Amitriptyline
d. All of the above d. Tranylcypromine
61–5. What is the most common mood disorder? 61–9. Which of the following selective serotonin-reuptake
a. Schizophrenia inhibitors has been most closely associated with fetal
b. Bipolar disorder heart defects?
c. Major depression a. Citalopram
d. Anorexia nervosa b. Fluoxetine
c. Paroxetine
d. Sertraline
Psychia tric Disorders 397

61–10. A 25-year-old G1P0 at 12 weeks’ gestation with a 61–15. Lithium has been linked to which of the following
history of major depression presents for prenatal fetal anomalies?
care. She reports that she was taking paroxetine a. Club foot
up until 2 weeks ago when she found out she b. Omphalocele
was pregnant. Her psychiatrist has changed her
medication, and she is doing well. Based on her
c. Ebstein anomaly
history, you should consider offering her which of d. Pulmonary sequestration
the following?
a. Amniocentesis 61–16. Which of the following statements regarding
b. Pregnancy termination postpartum psychosis is true?
c. Fetal echocardiography a. It is more common in multiparas.
d. Maternal brain magnetic resonance imaging b. Its incidence is 1/10,000 deliveries.
c. It usually manifests 6–8 weeks after delivery.
61–11. A postpartum patient who is exclusively breast d. It is more common in patients with obstetrical
feeding is diagnosed with major depression. The complications.
decision is made to start a selective serotonin-
reuptake inhibitor. Which of the following has the 61–17. What is the most important risk factor for
highest detectable concentrations in breast milk? postpartum psychosis?
a. Citalopram a. Drug abuse
b. Fluoxetine b. Bipolar disorder
c. Paroxetine c. Major depression
d. Sertraline d. Multifetal gestation

61–12. Pregnant women not adequately prepared for 61–18. What is the recurrence risk of postpartum psychosis?
electroconvulsive therapy are at increased for all a. 5%
EXCEPT which of the following?
b. 10%
a. Aspiration
c. 25%
b. Hypotension
d. 50%
c. Respiratory acidosis
d. Aortocaval compression 61–19. A 32-year-old G4P3 at 39 weeks’ gestation presents
in active labor. The patient had postpartum
61–13. Which of the following statements regarding bipolar psychosis in her last pregnancy. In addition to close
disorder is true? monitoring for recurrence and involvement of the
a. Periods of depression last at least 2 months. family, you plan for her to initiate which of the
b. There is no genetic component to the illness. following agents in the puerperium?
a. Lithium
c. Up to 20% of patients with this illness commit
suicide. b. Fluoxetine
d. It is more common among pregnant women c. Citalopram
compared with nonpregnant women. d. No pharmacologic agent

61–14. A 25-year-old G1P0 at 18 weeks’ gestation presents 61–20. Which of the following statements regarding anxiety
for prenatal care. The patient has a long history disorders is true?
of debilitating bipolar disorder. She had been a. They are more common in pregnant women than
prescribed lithium and was taking this during the nonpregnant women.
early weeks of her current pregnancy. Based on her
b. The prevalence of these disorders in adults in the
history, you should offer her which of the following?
United States is 1.8%.
a. Amniocentesis
c. With generalized anxiety disorder, symptoms and
b. Pregnancy termination severity increase across pregnancy.
c. Fetal echocardiography
d. They are often characterized by irrational fear,
d. Maternal brain magnetic resonance imaging nausea, insomnia, dizziness, and frequent
urination.
398 Medical and Surgica l Complications

61–21. A 21-year-old G1P0 at 15 weeks’ gestation presents 61–25. A 30-year-old G2P1 at 8 weeks’ gestation with a
for prenatal care. She has a history of anxiety and history of schizophrenia controlled with haloperidol
is concerned for her baby. Which of the following presents for prenatal care. Which medication
counseling points are true? regimen do you have planned for her during
a. No pharmacotherapy agents can be used safely in pregnancy?
pregnancy. a. Continue her current medication regimen
b. Pharmacotherapy for anxiety has been strongly b. Stop haloperidol until the second trimester
linked to cleft lip and palate. c. Change haloperidol to an atypicalantipsychotic
c. Benzodiazepines taken during the third trimester d. Stop haloperidol for the remainder of pregnancy
can cause neonatal withdrawalsyndrome. and manage her with psychotherapy
d. All studies have shown that her child is at
increased risk for various neuropsychiatric 61–26. Obstetrical complications of eating disorders include
conditions. which of the following?
a. Spontaneous abortion
61–22. Which of the following is not a prominent feature of b. Low neonatal birthweight
schizophrenia spectrum disorders?
c. Poor maternal wound healing
a. Somnolence d. All of the above
b. Hallucinations
c. Disorganized thinking 61–27. Which of the following statements regarding eating
d. Abnormal motor behavior disorders is true?
a. Lifetime prevalence is 10–20%.
61–23. If one parent has schizophrenia, what is the risk of b. Eating disorder symptoms worsen with
this condition developing in offspring? pregnancy.
a. < 1% c. Binge-eating disorder is associated with low
b. 5–10% neonatal birthweight.
c. 15–20% d. Pregnant women with a history of an eating
d. 50–60% disorder should be closely monitored for weight
gain.
61–24. Within 5 years from the first signs of schizophrenia,
what percent of patients are employed? 61–28. Which of the following is not a personality disorder?
a. 10% a. Paranoid
b. 30% b. Schizoid
c. 50% c. Schizotypal
d. 70% d. Schizoaffective
406

Neoplastic Disorders

63–1. Which o the ollowing is the most common benign 63–7. An ovarian cancer survivor presents or
neoplasm in pregnancy? preconceptional counseling. She has had prior pelvic
a. Ovarian cyst irradiation. With this history, she is at increased risk
b. Pyogenic granuloma or all EXCEPT which o the ollowing obstetrical
complications?
c. Endocervical polyp
a. Stillbirth
d. Breast ibroadenoma
b. Preterm birth
63–2. What are the most common cancers in pregnancy? c. Fetal birthweight < 2500 g
a. Breast, thyroid, cervix d. Fetal congenital mal ormations
b. Breast, cervix, lymphoma
63–8. Published case reports describe rare instances o
c. Breast, thyroid, lymphoma maternal tumor metastasizing to the etus. With
d. hyroid, cervix, melanoma which o the ollowing cancer types does this most
requently occur?
63–3. O the ollowing imaging modalities, which is the a. Melanoma
sa est during pregnancy?
b. Leukemia
a. Sonography
c. Lymphoma
b. Computed tomography
d. Breast cancer
c. Diagnostic radiography
d. Magnetic resonance imaging 63–9. An asymptomatic 19-year-old G1P0 presents or
prenatal care. Her routine obstetrical care should
63–4. When radiation therapy is needed during pregnancy, include which o the ollowing elements?
which o the ollowing potential etal risks should a. Pelvic examination alone
the patient be in ormed o ?
b. Pelvic examination and Pap smear
a. Microcephaly
c. Pelvic examination, Pap smear, and human
b. Mental retardation papillomavirus testing
c. Growth restriction
d. Pelvic examination, Pap smear, and human
d. All o the above papillomavirus vaccination

63–5. Embryonic exposure to cytotoxic drugs may cause 63–10. For at-risk women, cervical cytological screening
major congenital mal ormations in what percentage is recommended more requently than the routine
o cases? screening guideline schedule. Which o the ollowing
a. 2% patients should have more requent cervical cancer
b. 20% screening?
c. 33% a. 33-year-old with history o diabetes
d. 50% b. 19-year-old with three li etime sexual partners
c. 22-year-old with in utero diethylstilbestrol
63–6. Why is chemotherapy occasionally withheld in the exposure
last 3 weeks prior to delivery? d. 26-year-old with prior human papillomavirus
vaccination
a. o allow the patient to breast eed
b. o decrease the likelihood o maternal neutropenia
c. o decrease the chance o etal-growth restriction
d. o decrease the overall risk o late mutagenic e ects
Neopla stic Disorders 407

63–11. Which o the ollowing serovars are considered high- 63–17. A 29-year-old G2P1 patient at 22 weeks’ gestation
risk human papillomavirus serotypes? is re erred to your clinic or colposcopy due to
a. 6, 11 a Pap smear report noting high-grade squamous
intraepithelial lesion (HSIL). What can you tell her
b. 6, 16
regarding colposcopy in pregnancy?
c. 11, 18 a. It is best delayed to the puerperium.
d. 16, 18 b. It is more likely to be unsatis actory.
c. She will need a repeat study at 26 weeks’ gestation.
63–12. A 40-year-old pregnant patient calls to discuss her
Pap smear results. Her Pap smear shows atypical d. It is indicated or the evaluation o HSIL lesions.
squamous cells o undetermined signi icance, and
human papillomavirus testing is negative. Which 63–18. he cervical biopsy rom the patient in Question
o the ollowing is your recommendation or 63–17 shows invasive squamous cell carcinoma.
subsequent evaluation? Which o the ollowing is a part o ormal cervical
a. Colposcopy 6 weeks postpartum staging in pregnancy?
b. Colposcopy in the third trimester a. Chest radiograph
c. Repeat cytology 6 weekspostpartum b. Renal sonography
d. Repeat cytology in the third trimester c. Abdominopelvic computed tomography
d. None o the above
63–13. For a cervical intraepithelial neoplasia (CIN) 1 lesion
diagnosed in early pregnancy, what is the chance 63–19. I microinvasive disease is diagnosed rom a cervical
that repeat cytology at the end o the puerperium conization specimen obtained in the early second
will be normal? trimester, which o the ollowing statements is true
a. 20–30% regarding pregnancy management?
b. 40–50% a. She may deliver vaginally.
c. 60–70% b. She will need cesarean delivery.
d. 80–90% c. She will require adjuvant radiotherapy in the
puerperium.
63–14. Cervical conization during pregnancy is associated d. She will need radical hysterectomy at the time o
with an increased risk o all EXCEPT which o the delivery.
ollowing?
a. Preterm delivery 63–20. All EXCEPT which o the ollowing are reasons
b. Membrane rupture that surgical treatment is pre erred to radiotherapy
in the treatment o early-stage pregnancy-associated
c. Residual neoplasia
invasive cervical cancer?
d. Inadequate diagnostic tissue sample
a. Radiation therapydestroys ovarian unction.
b. Radiation therapy can cause intestinal injury.
63–15. What is the incidence o invasive cervical cancer in
c. Complication rates are higher with radiotherapy.
the general obstetrical population?
d. Surgical treatment is more e ective than
a. 1 in 2500
radiotherapy.
b. 1 in 5500
c. 1 in 8500 63–21. A 32-year-old nulligravida requests ertility-sparing
d. 1 in 11,500 radical trachelectomy or her small Stage IB1 cervical
cancer. How should you counsel her regarding this
63–16. What percentage o cervical cancers are treatment option and uture pregnancy outcomes?
adenocarcinomas? a. Her risk o premature delivery is decreased.
a. 5% b. She will need cesarean delivery in a uture
b. 20% pregnancy.
c. 33% c. Her ertility remains the same as thegeneral
d. 50% obstetrical population.
d. Her chance o a live birth would be the same as
the general obstetrical population.
408 Medical and Surgica l Complications

63–22. You elect to per orm transvaginal sonography or 63–24. What is the recommended standard o care or the
your patient with a uterus that is larger than treatment o endometrial carcinoma?
expected based on her gestational age. Her sonogram a. Curettage with progestational treatment
is shown here. Regarding leiomyomas in pregnancy, b. Curettage without progestational treatment
which o the ollowing statements is true?
c. otal abdominal hysterectomy and bilateral
salpingoophorectomy
a. hey require sonographic surveillance. d. All o the above
b. heir incidence in pregnancy approximates 2%.
c. hey grow in pregnancy due to human chorionic 63–25. A 17-year-old gravida is re erred or evaluation o
gonadotropin stimulation. an adnexal mass ound at 9 weeks’ gestationduring
sonography or irst-trimester bleeding. A sonogram
d. he vaginal delivery rate is only 30% i o her right ovary is shown, and dimensions o
leiomyomas are larger than 10 cm. the mass are marked by calipers. Based on its
appearance, what is the most likely diagnosis?

63–23. Prior to conception, uterine artery embolization


(UAE) was recommended or your patient with a. Serous cystadenoma
symptomatic leiomyomas. Rates o which o the b. Corpus luteum cyst
ollowing are increased in pregnancies ollowing
UAE? c. Mature cystic teratoma
a. Miscarriage d. Ovarian hyperstimulation syndrome
b. Cesarean delivery
c. Postpartum hemorrhage
d. All o the above
Neopla stic Disorders 409

63–26. A pregnant patient at 11 weeks’ gestation complains 63–27. A patient at 9 weeks’ gestation is treated or
o intermittent le t lower quadrant pain and nausea a ruptured and excised corpus luteum cyst o
and vomiting. She undergoes diagnostic exploratory pregnancy. Which o the ollowing regimens is
laparotomy. Which o the ollowing is true regarding recommended?
management, in general, o patients with this a. Micronized progesterone, 200 mg orally daily or
diagnosis? 3 weeks
b. 17-hydroxyprogesterone caproate, 150 mg
intramuscularly × 1 dose
a. Excision o the adnexa is alwaysrequired.
c. 8% progesterone vaginal gel, one applicator
b. All patients require progesterone replacement. vaginally daily or 1 week
c. Oophoropexy could prevent a recurrence o this d. 17-hydroxyprogesterone caproate, 150 mg
complication. intramuscularly weekly or 3 weeks
d. All o the above
63–28. Which o the ollowing is a rare, solid benign
ovarian tumor that can virilize a emale etus?
a. Luteoma
b. Brenner tumor
c. Mature cystic teratoma
d. Hyperreactio luteinalis

63–29. A pregnant patient presents with bilateral adnexal


masses. She is pregnant with twins. Both masses are
similar to the image shown below. What is the most
likely diagnosis?

a. Luteoma
b. Granulosa cell tumor
c. Mature cystic teratoma
d. Hyperreactio luteinalis
410 Medical and Surgica l Complications

63–30. Which o these pregnancy-associated adnexal 63–36. All EXCEPT which o the ollowing may be
masses can lead to hypovolemia with ascites, renal indicated in the management o thyroid cancer in
dys unction, and adult respiratory distress syndrome? pregnancy?
a. Luteoma a. Radioiodine
b. Corpus luteum b. Delay surgery until the second trimester
c. Granulosa cell tumor c. Expectant management with thyroxine treatment
d. Hyperstimulated ovary d. Surgical treatment with thyroxine replacement

63–31. What percentage o ovarian cancer ound in 63–37. Reed-Sternberg cells, such as the one shown here, are
pregnancy is early-stage? consistent with which o the ollowing lymphoid cell
a. 25% malignancies?
b. 50%
c. 75% a. Hodgkin lymphoma
d. 90% b. Hairy cell leukemia
c. Acute myeloid leukemia
63–32. All EXCEPT which o the ollowing are considered d. cell derived lymphoma
risk actors or breast cancer?
a. Nulliparity 63–38. What is the system used or staging Hodgkin and
b. Advancing age other lymphomas called?
c. Breast eeding a. Clark
d. BRCA1 and BRCA2 gene mutation b. Breslow
c. Bethesda
63–33. A pregnant patient at 26 weeks’ gestation has a d. Ann Arbor
palpable breast mass and undergoes mammography
and needle biopsy o the identi ied lesion. Although 63–39. What standard single-agent chemotherapy regimen
clinical examination and biopsy resultssuggest is recommended or treatment o Hodgkin disease
a benign mass, mammogram indings suggest during the irst trimester?
malignancy. What should be the next management
step? a. Bleomycin
a. Chemotherapy b. Vinblastine
b. Mass excision c. Dacarbadine
c. Repeat mammography d. Doxorubicin
d. Repeat needlebiopsy

63–34. Your evaluation o the patient in Question 63–33


con irms breast cancer in pregnancy. For women
with this diagnosis, all EXCEPT which o the
ollowing treatments are associated with improved
patient survival rates?
a. Chemotherapy
b. otal mastectomy
c. Modi ied mastectomy
d. Pregnancy termination

63–35. he patient in Question 63–33 is now 28 weeks


pregnant. Her axillary nodes were positive or cancer,
and chemotherapy is indicated. All EXCEPT which
o the ollowing agents are acceptable in pregnancy?
a. rastuzumab
b. Doxorubicin
c. 5-Fluorouracil
d. Cyclophosphamide
Neopla stic Disorders

63–40. Which virus is associated with Burkitt lymphoma? 63–42. A patient presents at 14 weeks’ gestation with the
a. Hepatitis C skin lesion shown below and palpable lymph nodes.
b. Herpesvirus Which o the ollowing dermatological neoplasms is
this is consistent with?
c. Epstein-Barr virus
d. Human immunode iciency virus Reproduced wit permission by Hardin MJ: Cutaneous conditions. In Knoop KJ, Stack
LB, Storrow AB, et al (eds): T e Atlas o Emergency Medicine, 3rd ed. New York,
McGraw-Hill, 2010, Figure 13-80.
63–41. Which o the ollowing statements regarding
leukemia in pregnancy is alse?
a. Stage I melanoma
a. Leukemias may arise rom bone marrow.
b. Stage II melanoma
b. Remission is common duringpregnancy.
c. Stage I basal cell carcinoma
c. ermination o pregnancy will improve
prognosis.
d. Stage II squamous cell carcinoma
d. Leukemias are more common in patients older
than 40 years.

CHAPTER00
InfectiousDiseases

64–1. Which o the ollowing is the primary 64–4. A 26-year-old G1P0 at 32 weeks’ gestation is seen in
immunological etal response to in ection? your o ice or ever, myalgias, and a rash o 2 days’
a. IgA duration. She has widely dispersed vesicular lesions
b. IgE that are pruritic. A picture o her ace is provided
here. Mortality rom this in ection is predominately
c. IgG due to which o the ollowing?
d. IgM
Reproduced with permission rom Wolf K, Johnson RA (eds): Viral in ections o skin and
64–2. What percentage o women who have not previously mucosa. In Fitzpatrick’s Color Atlas and Synopsis o Clinical Dermatology, 6th ed. New
had varicella will become in ected a ter exposure? York, McGraw-Hill, 2009, Figure 27-38.

a. 1–2%
a. Pneumonia
b. 5–10%
b. Meningitis
c. 20–30%
c. Liver ailure
d. 60–95% d. Renal ailure
64–3. Which o the ollowing clinical intervals best
describes the contagious period or varicella?
a. From rash appearance until lesion crusting
b. From varicella exposure until lesion appearance
c. From 1 day be ore rash onset until lesion crusting
d. From rash appearance until complete lesion
resolution
414 Medical and Surgica l Complications

64–5. Which o the ollowing statements regarding herpes 64–7. An 18-year-old G1P0 at 36 weeks’ gestation presents
zoster, shown here, is true? in active labor and appears to have active varicella.
Despite all e orts, the in ant delivers vaginally
shortly therea ter. You are very concerned or which
o the ollowing reasons?
a. It is painless.
a. Maternal antibody has not yet ormed.
b. It is contagious i blisters are broken.
b. Mortality rates or the newborn approach 30%.
c. It is responsible or many congenital
c. The in ant may develop disseminated visceral and
mal ormations.
central nervous system disease.
d. It is more requent and more severe in pregnant
d. All o the above
women.

64–8. A 19-year-old G2P0 at 37 weeks’ gestation presents


or her routine prenatal care appointment. While
in the waiting room, she has a conversation with
64–6. One mani estation o congenital varicella syndrome another patient who is coughing and scratching at a
is demonstrated in the photograph below. At which vesicular rash. The 19-year-old denies a prior history
o the ollowing gestational ages did in ection most o chickenpox, and you are now concerned that she
likely occur? has been exposed to the varicella-zoster virus (VZV).
What should be done or her now?
Reproduced with permission rom Cunningham FG, Leveno KJ, Bloom SL, et al (eds): In a. O er varicella vaccination with Varivax
ectious diseases. In Williams Obstetrics, 24th ed. New York, McGraw-Hill, 2014, Figure b. De er VZV serology testing and proceed with
64-1.
varicella zoster immune globulin (VariZIG)
administration
a. < 6 weeks
c. Test or VZV serology and administer VariZIG i
b. 6–10 weeks results are negative or antibodies to VZV
c. 13–20 weeks
d. Test or VZV serology and give VariZIG and
d. 28–32 weeks Varivax i results are negative or antibodies to VZV

64–9. Which o the ollowing statements regarding the


varicella vaccine, Varivax, is alse?
a. Seroconversion is98%.
b. It is not secreted into breast milk.
c. It is recommended in pregnancy to all women
with no history o varicella.
d. At 10 years postvaccination, the breakthrough
in ection rate approximates5%.

64–10. During which o the ollowing periods should


in luenza vaccination be avoided?
a. First trimester
b. Postpartum, i breast eeding
c. During pregnancy and or 1 month be ore
becoming pregnant
d. None o the above
Infectious Disea ses 415

64–11. A 22-year-old G1P0 at 32 weeks’ gestation presents 64–12. A 31-year-old G1P0 at 8 weeks’ gestation presents
with ever, coryza, conjunctivitis, cough, and rash. complaining o a ever and rash o 5 day’s duration.
The patient was raised in a separatist group that does The rash startedon her ace and has spread to her
not believe in immunizations. You suspect measles, trunk and extremities. A photograph o her chest
and a photograph o her ace is provided here. is provided here. Given her gestational age, which
Which o the ollowing counseling points is true? o the ollowing in ections would be the most
concerning or this patient and her etus?

a. Treatment is supportive.
a. Rubeola
b. The virus is not teratogenic.
b. Rubella
c. There is an increased incidence o preterm
delivery and low birthweight with maternal c. Varicella
in ection. d. Toxoplasmosis
d. All o the above
64–13. I the patient in Question 64–12 has acute rubella,
you would expect serologic testing to most likely
show which o the ollowing results?
a. IgM positive, IgG negative
b. IgM negative, IgG negative
c. IgM negative; IgG positive, high a inity
d. IgM positive; IgG positive, high a inity

64–14. With maternal rubella in ection at 8 weeks’


gestation, the risk o congenital in ection most
closely approximates which o the ollowing?
a. 1%
b. 10%
c. 25%
d. up to 90%
416 Medical and Surgica l Complications

64–15. The most common single de ect associated with 64–18. For the patient in Question 64–17, you obtain and
congenital rubella is which o the ollowing? send serological testing or parvovirus B19. Which
a. Glaucoma o the ollowing testing outcomes would satis y you
b. Microcephaly that she is immune and needs no urther evaluation?
c. Sensorineural dea ness a. IgG positive, IgM negative
d. Pulmonary artery stenosis b. IgG positive, IgM positive
c. IgG negative, IgM negative
64–16. Hand oot and mouth disease is caused by which o d. IgG negative, IgM positive
the ollowing viruses?
a. Togavirus 64–19. The serological test results or the patient in
b. Parvovirus Question 64–17 indicate that she has had a recent
parvovirus B19 in ection. Your management plan at
c. Coxsackievirus
this point should be which o the ollowing?
d. None o the above
a. O er pregnancy termination
b. Begin serialmeasurement o maternal viral loads
64–17. A 20-year-old G1P0 at 18 weeks’ gestation presents every 2 weeks or 10 weeks
to your o ice or prenatal care. The patient just
started working at a day-care center and is concerned c. Begin serial etal sonographic evaluation every
about acquiring in ections rom the children. She 2 weeks or the irst 10 weeks a ter in ection
reports several colds since starting the job. She d. Per orm sonographic etal evaluation now to
wasn’t eeling well last week and noted headache, identi y hydrops and, i the scan results are
nausea, and pharyngitis. Three weeks ago, one normal, resume prenatal care with no urther
child in particular looked rather ill. She shows you sonographic surveillance
a picture o the child on her cell phone, and the
photograph is provided here. Your concern or this 64–20. Four weeks a ter irst presenting or care, the etus
patient is in ection with which o the ollowing? o the patient in Question 64–17 is now hydropic
and shows elevated middle cerebral artery peak
systolic velocities. You counsel her regarding etal
a. Parvovirus
blood sampling and etal trans usion. Which o the
b. Rubeola virus ollowing is a true statement?
c. Rubella virus a. Most etuses require multiple trans usions.
d. Coxsackievirus b. The mortality rate without trans usion is 90%.
c. The overall mortality rate ollowing trans usion is
50%.
d. Reports o the neurodevelopmental outcomes o
etuses trans used or parvovirus-induced anemia
are con licting.

64–21. Which o the ollowing statements regarding


cytomegalovirus (CMV) is true?
a. Pregnancy increases the severity o maternal
CMV in ection.
b. It is the most common perinatal in ection in the
developed world.
c. Most women who contract CMV will have
symptoms that will alert the provider.
d. Once a woman has been in ected with CMV, she
is immune and will never become ill again rom
CMV.
Infectious Disea ses 417

64–22. Which o the ollowing is not a risk actor 64–27. A 19-year-old G1P0 at 29 weeks’ gestation presents
or intrapartum group B streptococcus (GBS) to your o ice complaining o lulike symptoms and
transmission? a target-shaped rash on her neck, which is shown in
a. Membranes ruptured ≤ 18 hours this photograph. She has just returned rom a hiking
b. Delivery at < 37 weeks’ gestation trip in New Hampshire. Her history and rash are
concerning or which o the ollowing?
c. Intrapartum temperature ≥ 38.0o Celsius
d. Prior in ant with invasive early-onset GBSdisease
a. Lyme disease
64–23. Which o the ollowing antibiotics is never b. Toxoplasmosis
an appropriate choice or penicillin-allergic c. West Nile virus in ection
patients requiring group B streptococcus (GBS) d. Methicillin-resistant Staphylococcus aureus
chemoprophylaxis? in ection
a. Ce azolin
b. Vancomycin
c. Clindamycin 64–28. Prompt treatment o the patient in Question 64–27
should prevent most adverse pregnancy outcomes.
d. Erythromycin
Assuming she has no allergies, which o the
ollowing is the pre erred treatment?
64–24. Which o the ollowing is irst-line inpatient therapy a. Spiramycin
or methicillin-resistant Staphylococcus aureus
in ection?
b. Vancomycin
a. Ri ampin c. Amoxicillin
b. Vancomycin d. Doxycycline
c. Clindamycin
64–29. Which o the ollowing statements regarding
d. Trimethoprim-sul amethoxazole
toxoplasmosis in ection in pregnancy is true?
a. The risk o etal in ection increases with
64–25. A 25-year-old G3P2 at 34 weeks’ gestation presents
gestational age.
with decreased etal movement, ever, myalgias,
and headache or 4 days. Her etus is dead. The b. The risk o etal in ection decreases with
patient’s cervix is 3-cm dilated, and with rupture o gestational age.
membranes, dark-brown luid is seen. Postpartum, c. The severity o etal in ection is much greater in
the patient is diagnosed with listeriosis. Which o the late pregnancy.
ollowing is pre erred treatment or her? d. The risk and severity o etal in ection are not
a. Vancomycin dependent on gestational age.
b. Ce triaxone
c. Erythromycin
d. Ampicillin and gentamicin

64–26. Which o the ollowing is not a typical sign or


symptom o salmonellosis?
a. Fever
b. Abdominal pain
c. Bloody diarrhea
d. Nausea/vomiting
418 Medical and Surgica l Complications

64–30. Which o the ollowing etal outcomes is not 64–32. Which o the ollowing statements regarding
associated with toxoplasmosis in ection during prenatal screening or toxoplasmosis is alse?
pregnancy? a. Screening is recommended in areas o low
a. Hydrocephalus prevalence.
b. Chorioretinitis b. The parasite is rarely detected in tissue or
c. Placental microabscesses body luids.
d. Intracranial calci ications c. IgM antibodies alone should not be used to
detect acute toxoplasmosis.
64–31. A di erential diagnosis or the indings seen in this d. The presence o high-avidity IgG indicates that
etal sonographic image would include all EXCEPT in ection did not occur in the preceding 3 to 5
which o the ollowing? months.

64–33. For a pregnant woman with chloroquine-resistant


malaria, the current recommended treatment is
a. Rubella which o the ollowing?
b. Toxoplasmosis a. Me loquine
c. Parvovirus in ection b. Atovaquone
d. Cytomegalovirus in ection c. Quinine and erythromycin
d. Doxycycline and primaquine

64–34. Which o the ollowing statements regarding West


Nile virus is true?
a. Treatment is with acyclovir.
b. The incubation period is 3 to 4 weeks.
c. O those who are in ected, 25% develop
meningoencephalitis or acute laccid paralysis.
d. Diagnosis is by detection o viral IgG and IgM in
serum and IgM in cerebrospinal luid.

64–35. I a pregnant woman develops anthrax and has


li e-threatening allergies to both cipro loxacin and
penicillin, which o the ollowing is a recommended
alternative?
a. Ri ampin
b. Vancomycin
c. Doxycycline
d. Trimethoprim-sul amethoxazole
420

Sexuall Transmitted Infections

65–1. Which of the following does not increase the risk of 65–4. The best test to perform for the patient in Question
transmission ofsyphilis? 65–3 to ascertain a definitive diagnosis is whichof
a. Cervical inversion the following?
b. Cervical hyperemia a. Rapid plasma reagin (RPR)
c. Cervical friability b. Bacterial culture of lesion exudate
d. Abrasions of the vaginal mucosa c. Dark-field examination of lesion exudate
d. Serum assay for herpes simplex virus 1 and
65–2. The incubation period of syphilis is which of the 2 antibodies
following?
a. 1–7 days 65–5. The lesions in this photograph are most indicative of
b. 10 days which stage of syphilis?
c. 3–90 days Used with permission from Dr. Jonathan Willms.
d. 120–180 days
a. Primary
65–3. A 21-year-old G4P2 at 17 weeks’ gestation presents b. Secondary
for her first prenatal care visit. She has a history c. Late latent
of prostitution, but she denies engaging in such
d. Early latent
activities for the past month. During examination, a
painless lesion is noted on the right labia. A picture
65–6. Findings of secondary syphilis include all EXCEPT
of the lesion is provided below. The most likely
which of the following?
diagnosis is which of the following?
a. Fever
Reproduced with permission from Cunningham FG, Leveno KJ, Bloom SL, et al (eds):
b. Macular rash
Sexually transmitted infections. In Williams Obstetrics, 24th ed. New York, McGraw-
Hill, 2014, Figure 65-1. c. Condylomata lata
d. Strawberry cervix
a. Chancroid
b. Primary syphilis
c. Bartholin gland duct abscess
d. Herpes simplex virus infection
Sexua lly Tra nsmitted Infections 421

65–7. A 40-year-old G9P8 at 10 weeks’ gestation is 65–12. In women with gonorrhea, what percent also have
found to have a rapid plasma reagin (RPR) of 1:4. chlamydial infection?
Confirmatory testing is positive. She has no signs a. 1%
or symptoms of syphilis. The patient’s RPR was
b. 40%
nonreactive in all of her prior pregnancies, and she
had a nonreactive RPR 6 months ago. Her stage is c. 75%
which of the following? d. 100%
a. Primary
b. Secondary 65–13. Risk factors for gonorrhea that should prompt
screening in pregnancy include all EXCEPT which
c. Late latent
of the following?
d. Early latent
a. Drug use
b. Age > 25 years
65–8. A 20-year-old G1P0 at 10 weeks’ gestation presents
c. New sexual partner
to your office for a rash of her palms and soles. She
saw an informational poster on syphilis, and she is d. Prior gonococcal infection
concerned that she has it. Her RPR titer is 1:64,
and confirmatory testing is positive. The appropriate 65–14. The current recommendation for the treatment of
treatment is which of the following? uncomplicated gonococcal infection in pregnancy is
a. Benzathine penicillin G, 2.4 million units which of the following?
intramuscularly, weekly for two doses a. Azithromycin 1 g orally as a single dose
b. Benzathine penicillin G, 2.4 million units b. Ceftriaxone 250 mg intramuscularly as a single
intramuscularly, weekly for three doses dose
c. Benzathine penicillin G, 2.4 million units c. Erythromycin ethylsuccinate 400 mg orally four
intramuscularly, weekly for four doses times daily for 14 days
d. Aqueous procaine penicillin, 2.4 million units d. Ceftriaxone 250 mg intramuscularly as a single
intramuscularly, plus probenecid 500 mg orally dose plus azithromycin 1 g orally as a single dose
four times daily, both for 10 days
65–15. A 15-year-old G1P0 presents at 18 weeks’ gestation
65–9. The patient in Question 65–8 does not come to any complaining of pain of her wrists, the tops of her
of her subsequent appointments. Now at 36 weeks’ hands, ankles, and the tops of her feet. She has never
gestation, she represents, you repeat her RPR, and it experienced anything like this previously. She has no
is 1:128. The most likely reason for this is which of medical problems and eight lifetime sexual partners.
the following? A nucleic acid amplification test is positive for
a. Laboratory error gonorrhea, and you suspect disseminatedinfection.
The treatment for this is which of the following?
b. Successful treatment
a. Azithromycin 1 g orally as a single dose
c. Reinfection or treatment failure
d. Coexistent systemic lupus erythematosus b. Ceftriaxone 250 mg intramuscularly as a single
dose
c. Ceftriaxone 250 mg intramuscularly as a single
65–10. A pregnant woman with syphilis and a true
dose plus azithromycin 1 g orally as a single dose
penicillin allergy should be treated with which of the
following? d. Ceftriaxone 1000 mg intravenously every
24 hours continued for 24 to 48 hours after
a. Erythromycin
improvement followed by oral therapy to
b. Azithromycin complete a week of treatment
c. Doxycycline, after delivery
d. Benzathine penicillin G, after desensitization 65–16. With 1.4 million cases reported in 2011, which
of the following is the most commonly reported
65–11. All EXCEPT which of the following are infectious disease in the United States?
manifestations of a Jarisch-Herxheimer reaction in a a. Chancroid
pregnant woman treated for syphilis? b. Gonorrhea
a. Contractions c. Genital herpes
b. Night sweats d. Chlamydia trachomatisinfection
c. Decreased fetal movement
d. Late fetal heart rate decelerations
422 Medical and Surgica l Complications

65–17. The most commonly identifiable infectious cause of 65–22. A 19-year-old G2P0 at 18 weeks’ gestation presents
ophthalmia neonatorum is which of the following? to the emergency room complaining of painful
a. Haemophilusducreyi lesions of the perineum. She reports fever at home.
b. Chlamydia trachomatis A photograph of the patient’s perineum is provided
below. The best next step in the management of this
c. Trichomonasvaginalis patient is which of the following?
d. Neisseria gonorrhoeae

65–18. First-line treatment for a chlamydial infection in a. Obtain lesion exudate for viral culture
pregnancy is which of the following?
b. Send human immunodeficiency virus (HIV)
a. Azithromycin 1000 mg orally as a single dose
testing
b. Ofloxacin 300 mg orally twice a day for 7 days
c. Diagnose patient with HSV by clinical
c. Doxycycline 100 mg orally twice a day for 7 days presentation alone
d. Erythromycin base 250 mg orally four times a d. Send spinal fluid for HSV polymerase chain
day for 14 days reaction (PCR) assay
65–19. What percentage of pregnant women acquire
65–23. At 36 weeks’ gestation, you elect to provide the
herpes simplex virus-1 (HSV-1) or HSV-2 during
patient in Question 65–22 acyclovir suppression.
pregnancy?
You do this for all EXCEPT which of the following
a. 0–0.1 reasons?
b. 0.5–2 a. It will decrease viral shedding.
c. 5–7.5 b. It reduces the chances of the newborn acquiring
d. 10–15 herpes to zero.
c. It will decrease the chance of her having an
65–20. The most common route of neonatal HSV outbreak at term.
transmission is which of thefollowing? d. It will decrease the chances of her requiring a
a. Postnatal cesarean delivery because of an outbreak.
b. Peripartum
c. Intrauterine
d. Preconceptional

65–21. A patient who has HSV-2 isolated from genital


secretions in the absence of HSV-1 or HSV-2
antibodies is classified as which of the following?
a. Transient infection
b. Reactivation disease
c. First episode primaryinfection
d. First episode nonprimaryinfection
Sexua lly Tra nsmitted Infections 423

65–24. A 27-year-old G2P1 at 39 weeks’ gestation presents 65–26. A 40-year-old G3P1 at 18 weeks’ gestation comes to
in active labor. The patient is known to have genital see you for a second opinion. A photograph of her
herpes, so a thorough exam is performed for lesions. perineum is provided below. She would like
No lesions are seen on the perineum or cervix, but treatment, but her current doctor refuses because he
a suspicious lesion is seen on her lower thigh.It feels the lesions will decrease in size after pregnancy.
is tender to the touch and suggestive of a herpetic Which of the following treatments would be
lesion. Your plan of management is which of the acceptable for use in pregnancy?
following? a. Imiquimod
a. Cesarean delivery b. Podophyllin resin
b. Acyclovir and tocolysis c. Interferon therapy
c. Occlusive dressing over the lesion and vaginal d. Trichloracetic acid
delivery
d. No change in management from any other 65–27. You see a 20-year-old G1P0 at 10 weeks’ gestation in
patient in labor preparing for a vaginal delivery your office for Gardasil exposure. The patient
reports that she received the first dose 1 month ago
65–25. After delivery of the patient in Question 65–24, she and before she knew she was pregnant. Your plan of
asks you if she can breast feed her infant given that care is which of the following?
she has active HSV. You counsel her which of the a. Continue theseries as any damage has already
following? been done
a. She should not breast feed. b. Delay the remaining doses until the patient has
b. She can breast feed, but only if she is not taking delivered
acyclovir. c. Delay the remaining doses until the patient has
c. She can breast feed while taking acyclovir but not delivered and completed her breast feeding
valacyclovir. d. Recommend termination of pregnancy because
d. She can breast feed, provided she has no breast of the strong likelihood of a significant fetal
lesions and practices strict hand washing. anomaly

65–28. Risk factors for bacterial vaginosis include all


EXCEPT which of the following?
a. Smoking
b. Douching
c. Vitamin D deficiency
d. Advanced maternalage
424 Medical and Surgica l Complications

65–29. A 15-year-old G1P0 at 15 weeks’ presents to the 65–31. Which of the following statements about the
emergency room complaining of foul-smelling, gray, treatment of HIV in pregnancy is true?
watery discharge. She also has pruritus. The results a. Treatment is recommended for all HIV-infected
of the wet mount are provided. The diagnosis is pregnant women.
which of the following?
b. All medications used for the treatment of HIV
are safe in pregnancy.
a. Gonorrhea
b. Candidiasis
c. Treatment is only required if the HIV-infected
woman would qualify for treatment when not
c. Trichomoniasis pregnant.
d. Bacterial vaginosis d. Zidovudine must be added to whatever regimen
the woman is already taking, even if her viral load
65–30. When is transmission of human immunodeficiency is adequately suppressed.
virus (HIV) from a woman to her baby most likely
to occur? 65–32. Which of the following statements regarding
a. Intrapartum intrapartum management of HIV is true?
b. Before 36 weeks’gestation a. If cesarean delivery is planned, it should be
c. In the days before delivery scheduled at 36 weeks’gestation.
d. Transmission occurs equally throughout gestation b. Cesarean delivery is recommended for women
with a viral load > 1000 copies/mL.
c. In labor, with a plan for vaginal delivery,
amniotomy should be performed as soon as
possible to hasten delivery.
d. In labor, internal monitors should be placed
because fetuses of HIV-infected women are at
increased risk for distress.

S-ar putea să vă placă și